Buy BOOKS at Discounted Price

Coordinates Geometry

Class 10th Mathematics KC Sinha Solution
Exercise 10.1
  1. In which quadrants do the following points lie:(10, -3)
  2. In which quadrants do the following points lie:(-4, -6)
  3. In which quadrants do the following points lie:(-8, 6)
  4. In which quadrants do the following points lie: ( {3}/{2} , 5 )…
  5. In which quadrants do the following points lie:(3, 0)
  6. In which quadrants do the following points lie:(0, -5)
  7. Plot the following points in a rectangular coordinate system:(4, 5)…
  8. Plot the following points in a rectangular coordinate system:(-2,-7)…
  9. Plot the following points in a rectangular coordinate system:(6,-2)…
  10. Plot the following points in a rectangular coordinate system:(-4, 2)…
  11. Plot the following points in a rectangular coordinate system:(4, 0)…
  12. Plot the following points in a rectangular coordinate system:(0, 3)…
  13. Where does the point having y-coordinate -5 lie?
  14. If three vertices of a rectangle are (-2, 0), (2, 0), (2, 1) find the…
  15. Draw the triangle whose vertices are (2, 3), (-4, 2) and (3, -1).…
  16. The base of an equilateral triangle with side 2a lies along the y-axis such…
  17. Let ABCD be a rectangle such that AB = 10 units and BC = 8 units. Taking AB and…
  18. ABCD is a square having a length of a side 20 units. Taking the centre of the…
Exercise 10.2
  1. (0, 0), (- 5, 12) Find the distance between the following pair of points:…
  2. (4, 5), (- 3, 2) Find the distance between the following pair of points:…
  3. (5, - 12), (9, - 9) Find the distance between the following pair of points:…
  4. (- 3, 4), (3, 0) Find the distance between the following pair of points:…
  5. (2, 3), (4, 1) Find the distance between the following pair of points:…
  6. (a, b), (- a, - b) Find the distance between the following pair of points:…
  7. Examine whether the points (1, - 1), (- 5, 7) and (2, 6) are equidistant from…
  8. Find a if the distance between (a, 2) and (3, 4) is 8.
  9. A line is of length 10 units and one of its ends is (- 2, 3). If the ordinate…
  10. Find the value of y for which the distance between the points P(2, - 3) and…
  11. (at1^2 , 2at1) and (at2^2 , 2at2) Find the distance between the points:…
  12. (a - b, b - a) and (a + b, a + b) Find the distance between the points:…
  13. (cosθ, sinθ) and (sinθ, cosθ) Find the distance between the points:…
  14. (7, 6) and (- 3, 4) Find the point on x - axis which is equidistant from the…
  15. (3, 2) and (- 5, - 2) Find the point on x - axis which is equidistant from the…
  16. (2, - 5) and (- 2, 9) Find the point on x - axis which is equidistant from the…
  17. Find the point on y - axis which is equidistant from point (- 5, - 2) and (3,…
  18. Find the point on y - axis which is equidistant from the points A(6, 5) and…
  19. (3, 5), (1, 1), (- 2, - 5) Using distance formula, examine whether the…
  20. (5, 1), (1, - 1), (11, 4) Using distance formula, examine whether the…
  21. (0, 0), (9, 6), (3, 2) Using distance formula, examine whether the following…
  22. (- 1, 2), (5, 0), (2, 1) Using distance formula, examine whether the following…
  23. (1, 5), (2, 3), (- 2, - 11) Using distance formula, examine whether the…
  24. If A = (6, 1), B = (1, 3) and C = (x, 8), find the value of x such that AB =…
  25. Prove that the distance between the points (a + rcosθ, b + rsinθ) and (a, b) is…
  26. use distance formula to show that the points (cosec^2 θ, 0), (0, sec^2 θ) and…
  27. Using distance formula show that (3, 3) is the centre of the circle passing…
  28. If the point (x, y) on the tangent is equidistant from the points (2, 3) and…
  29. Find a relation between x and y such that the point (x, y) is equidistant…
  30. If the distances of P(x, y) from points A(3, 6) and B(- 3, 4) are equal,…
  31. If the point (x, y) be equidistant from the points (a + b, b - a) and (a - b,…
  32. Prove that the points (3, 4), (8, - 6) and (13, 9) are the vertices of a right…
  33. (1, 1), (- root 3 , root 3), (- 1, - 1) Determine the type (isosceles, right…
  34. (0, 2), (7, 0), (2, 5) Determine the type (isosceles, right angled, right…
  35. (- 2, 5), (7, 10), (3, - 4) Determine the type (isosceles, right angled,…
  36. (4, 4), (3, 5), (- 1, - 1) Determine the type (isosceles, right angled, right…
  37. (1, 2 root 3), (3, 0), (- 1, 0) Determine the type (isosceles, right angled,…
  38. (0, 6), (- 5, 3), (3, 1) Determine the type (isosceles, right angled, right…
  39. (5, - 2), (6, 4), (7, - 2) Determine the type (isosceles, right angled, right…
  40. If A(at^2 , 2at), B (a/t^2 , 2a/t) and C(a, 0) be any three points, show that…
  41. If two vertices of an equilateral triangle be (0, 0) and (3, root 3), find the…
  42. Find the circum - centre and circum - radius of the triangle whose vertices…
  43. Find the centre of a circle passing through the points (6, - 6), (3, - 7) and…
  44. If the line segment joining the points A(a, b) and B(c, d) subtends a right…
  45. The centre of the circle is (2x - 1, 3x + 1) and radius is 10 units. Find the…
  46. Prove that the points (4, 3), (6, 4), (5, 6) and (3, 5) are the vertices of a…
  47. Prove that the points (4, 3), (6, 4), (5, 6) and (- 4, 4) are the vertices of…
  48. Prove that the points (3, 2), (6, 3), (7, 6), (4, 5) are the vertices of a…
  49. Prove that the points (6, 8), (3, 7), (- 2, - 2), (1, - 1) are the vertices of…
  50. Prove that the points (4, 8), (0, 2), (3, 0) and (7, 6) are the vertices of a…
  51. Show that the points A(1, 0), B(5, 3), C(2, 7) and D(- 2, 4) are the vertices…
  52. (4, 5), (7, 6), (4, 3), (1, 2) Name the type or quadrilateral formed, if any,…
  53. (- 1, - 2), (1, 0), (- 1, 2), (- 3, 0) Name the type or quadrilateral formed,…
  54. (- 3, 5), (3, 1), (0, 3), (- 1, - 4) Name the type or quadrilateral formed,…
  55. Two opposite vertices of a square are (- 1, 2) and (3, 2). Find the…
  56. If ABCD be a rectangle and P be any point in a plane of the rectangle, then…
  57. Prove, using co - ordinates that diagonals of a rectangle are equal.…
  58. Prove, using coordinates that the sum of squares of the diagonals of a…
Exercise 10.3
  1. Find the coordinates of the point which divides the line segment joining (2,4)…
  2. Find the coordinates of the point which divides the join of (-1,7) and (4,-3)…
  3. Find the coordinates of the point which divides the line segment joining the…
  4. Find the coordinates of the points which trisect the line segment joining the…
  5. Find the coordinates of the point of trisection of the line segment joining…
  6. The coordinates of A and B are (1,2) and (2,3) respectively, If P lies on AB,…
  7. If A (4,-8), B (3,6) and C(5,-4) are the vertices of a ΔABC, D is the…
  8. If p divides the join of A (-2,-2) and B (2,-4) such that ap/ab = 3/7 , find…
  9. A (1,4) and B (4,8) are two points. P is a point on AB such that AP = AB + BP.…
  10. The line segment joining A (2,3) and B(-3,5) is extended through each end by a…
  11. The line segment joining A(6,3) to B(-1,-4) is doubled in length by having half…
  12. The coordinates of two points A and B are (-1,4) and (5,1) respectively. Find…
  13. Find the distances of that point from the origin which divides the line segment…
  14. The coordinates of the middle points of the sides of a triangle are (1,1),…
  15. If the points (10,5),(8,4) and (6,6) are the mid-points of the sides of a…
  16. The mid-points of the sides of a triangle are (3,4),(4,6) and (5,7). Find the…
  17. A(1,-2) and B(2,5) are two points. The lines OA, OB are produced to C and D…
  18. Find the length of the medians of the triangle whose vertices are…
  19. If A(1,5), B (-2,1) and C(4,1) be the vertices of ΔABC and the internal…
  20. If the middle point of the line segment joining (3,4) and (k,7) is (x,y) and…
  21. one end of a diameter of a circle is at (2,3) and the center is (-2,5), find…
  22. Find the coordinates of a point A, where AB is the diameter of a circle whose…
  23. If the point C (-1,2) divides internally the line segment joining A (2,5) and…
  24. Find the ratio in which (-8,3) divides the line segment joining the points…
  25. In what ratio does the point (-4,6) divide the line segment joining the point…
  26. Find the ratio in which the line segment joining (-3,10) and (6,-8) is…
  27. Find the ratio in which the line segment joining (-3,-4) and (3,5) is divided…
  28. In what ratio does the x-axis divide the line segment joining the points…
  29. Find the ratio in which the line segment joining A(1,-5) and B(-4,5) is…
  30. Find the ratio in which the y-axis divides the line segment joining points…
  31. Find the centroid of the triangle whose vertices are (2,4), (6,4), (2,0).…
  32. The vertices of a triangle are at (2,2), (0,6) and (8,10). Find the…
  33. Two vertices of a triangle are (1,4) and (5,2). If its centroid is (0,-3),…
  34. The coordinates of the centroid of a triangle are (√3,2), and two of its…
  35. Find the centroid of the triangle ABC whose vertices are A (9,2), B(1,10) and…
  36. If (1,2), (0,-1) and (2,-1) are the middle points of the sides of the…
  37. Show that A(-3,2), B(-5,-5), C(2,-3) and D(4,4) are the vertices of a rhombus.…
  38. Show that the point (3,2),(0,5),(-3,2) and (0,-1) are the vertices of a…
  39. Prove that the points (-2,-1), (1,0),(4,3) and (1,2) are the vertices of a…
  40. Show that the points A(1,0), B(5,3), C(2,7) and D(-2,4) are the vertices of a…
  41. Prove that the point (4,8), (0,2), (3,0) and (7,6) are the vertices of a…
  42. Prove that the points (4,3), (6,4), (5,6) and (3,5) are the vertices of a…
  43. If (6,8), (3,7) and (-2,-2) be the coordinates of the three consecutive…
  44. Three consecutive vertices of a rhombus are (5,3), (2,7) and (-2,4). Find the…
  45. A quadrilateral has the vertices at the point (-4,2), (2,6), (8,5) and (9,-7).…
  46. If the points A(6,1), B(8,2), C(9,4) and D(p,3) are the vertices of a…
  47. Prove that the line segment joining the middle points of two sides of a…
  48. If P,Q,R divide the side BC,CA and AB of ΔABC in the same ratio, prove that…
Exercise 10.4
  1. (3, -4), (7, 5), (-1, 10) Find the area of the triangle whose vertices are…
  2. (-1.5, 3), (6, -2), (-3, 4) Find the area of the triangle whose vertices are…
  3. (-5, -1), (3, -5), (5, 2) Find the area of the triangle whose vertices are…
  4. (5, 2), (4, 7), (7, -4) Find the area of the triangle whose vertices are…
  5. (2, 3), (-1, 0), (2, -4) Find the area of the triangle whose vertices are…
  6. (1, -1), (-4, 6), (-3, -5) Find the area of the triangle whose vertices are…
  7. (at_1^2 , 2at_1) , (at_2^2 , 2at_2) , (at_3^2 , 2at_3) Find the area of the…
  8. (-5, 7), (-4, -5), (4, 5) Find the area of the triangle whose vertices are…
  9. (1, 1), (7, -3), (12, 2) and (7, 21) Find the area of the quadrilateral whose…
  10. (-4, 5), (0, 7), (5, -5), and (-4, -2) Find the area of the quadrilateral…
  11. Given (-5, 7), (-4, -5), (-1, -6) and (4, 5) Find the area of the…
  12. Given (0, 0), (6, 0), (4, 3), and (0, 3) Find the area of the quadrilateral…
  13. Given (1, 0), (5, 3), (2, 7) and (-2, 4) Find the area of the quadrilateral…
  14. Find the area of the quadrilateral whose vertices taken in order are (- 4, -2),…
  15. A median of a triangle divides it into two triangles of equal area. Verify this…
  16. If A, B, C are the points (-1, 5), (3, 1), (5, 7) respectively and D, E, F are…
  17. Three vertices of a triangle are A(1, 2), B(-3, 6) and C(5, 4). If D, E, and…
  18. Find the area of the triangle formed by joining the mid-points of the sides of…
  19. Find the area of a triangle ABC if the coordinates of the middle points of the…
  20. The vertices of ΔABC are A(3, 0), B(0, 6) and (6, 9). A straight line DE…
  21. If (t, t - 2), (t + 3, t) and (t + 2, t + 2) are the vertices of a triangle,…
  22. If A(x, y), B(1, 2) and C(2, 1) are the vertices of a triangle of area 6…
  23. Prove that the points (a, b+c), (b, c+a) and (c, a+b) are collinear.…
  24. If the points (x1y1), (x2, y2) and (x3, y3) be collinear, show that…
  25. If the points (a, b), (a1, b1) and (a-a1, b-b1) are collinear, show that a/a_1…
  26. Show that the point (a, 0), (0, b) and (1, 1) are collinear if 1/a + 1/b = 1…
  27. Find the values of x if the points (2x, 2x), (3, 2x+1) and (1, 0) are…
  28. Find the value of K if the points A(2, 3), B(4, k) and C(6, -3) are…
  29. Find the value of K for which the points (7, -2), (5, 1), (3, k) are…
  30. Find the value of K for which the points (8, 1), (k, -4), (2, -5) are…
  31. Find the value of P are the points (2, 1), (p, -1) and (-1, 3)collinear?…
  32. Show that the straight line joining the points A(0, -1) and B(15, 2) divides…
  33. ((a+1) (a+2), (a+2)), ((a+2) (a+3), (a+3)) and ((a, 3) (a+4), (a+4)) Find the…
  34. The point A divides the join of P(-5, 1) and Q(3, 5) in the ratio k:1. Find…
  35. The coordinates of A, B, C, D are (6, 3), (-3, 5), (4, -2) and (x, 3x)…
  36. If the area of the quadrilateral whose angular points taken in order are (1,…
  37. Find the area of the triangle whose vertices A, B, C are (3, 4) (-4, 3), (8,…
  38. The coordinates of the centroid of a triangle and those of two of its vertices…
  39. The area of a triangle is 3 square units. Two of its vertices are A(3,1),…
  40. The area of a parallelogram is 12 square units. Two of its vertices are the…
  41. Prove that the quadrilateral whose vertices are A(-2, 5), B(4, -1), C(9, 1)…
  42. Prove that points (-3, -1), (2, -1), (1, 1) and (-2, 1) taken in order are the…

Exercise 10.1
Question 1.

In which quadrants do the following points lie:

(10, -3)


Answer:


Given coordinate (10,-3) lies in Quadrant IV because as shown in the figure that those coordinate who have (+, -) sign lies in IV quadrants, or can say whose x-axis is “+” and y-axis is “–“ lies in IV Quadrant.



Question 2.

In which quadrants do the following points lie:

(-4, -6)


Answer:


Given coordinate (-4,-6) lies in Quadrant III because as shown in the figure that those points which have a sign like this (-, -) or can say whose both x-axis and y-axis is “–“ lies in III quadrants. So (-4,-6) lies in III Quadrant.



Question 3.

In which quadrants do the following points lie:

(-8, 6)


Answer:


Given coordinate (-8,6) lies in Quadrant II because as shown in the figure that those points which have a sign like this (-, +) lies in II quadrants. So (-8,6) lies in III Quadrant. Here also x-axis point is “-” and y-axis point is “+” so (-8,6) lies in Quadrant II.



Question 4.

In which quadrants do the following points lie:




Answer:


Given coordinate lies, in Quadrant I because as shown in the figure that those coordinate who have (+, +) sign lies in IV quadrants or can say whose x-axis is “+” and y-axis is also “+“ lies in I Quadrant.



Question 5.

In which quadrants do the following points lie:

(3, 0)


Answer:


Given coordinate is (3,0). This point lies on the x-axis because its y-axis is on origin. Therefore, it lies on the x-axis between the Quadrant I and Quadrant IV.



Question 6.

In which quadrants do the following points lie:

(0, -5)


Answer:


Given coordinate is (0, -5). This point lies on the y-axis because its x-axis is on origin. Therefore, it lies on the y-axis between the Quadrant III and Quadrant IV.



Question 7.

Plot the following points in a rectangular coordinate system:

(4, 5)


Answer:

Here is the graph for coordinate (4,5)




Question 8.

Plot the following points in a rectangular coordinate system:

(-2,-7)


Answer:



Question 9.

Plot the following points in a rectangular coordinate system:

(6,-2)


Answer:



Question 10.

Plot the following points in a rectangular coordinate system:

(-4, 2)


Answer:



Question 11.

Plot the following points in a rectangular coordinate system:

(4, 0)


Answer:



Question 12.

Plot the following points in a rectangular coordinate system:

(0, 3)


Answer:



Question 13.

Where does the point having y-coordinate -5 lie?


Answer:


The point having -5 lies on the on the y-axis on the negative side because here x-axis is 0 and when x-axis is 0 then points lies on the y-axis and when y-axis is 0 then point lies on the x-axis.


We can show it on graph with points (0, -5).



Question 14.

If three vertices of a rectangle are (-2, 0), (2, 0), (2, 1) find the coordinates of the fourth vertex.




Answer:

Here we have three vertices of rectangle say A (-2, 0) B (2, 0) and C (2, 1) so when we start graphing it on the graph as shown in the graph below then after plotting all three vertices you will get something like this.


Therefore, after joining all the vertices with a line segment, we will get our fourth vertex because in rectangle opposites sides are parallel and all the angles are right angle so, by joining all the lines according to properties of the rectangle you will get the fourth vertex. So fourth vertex of a rectangle is (-2, 1).




Question 15.

Draw the triangle whose vertices are (2, 3), (-4, 2) and (3, -1).


Answer:

It is easy to draw a triangle when it’s all vertices are given. We have to just locate all the given points on the graph and join them with a line as shown in the graph below.


Step 1. Locate all the vertices on the graph.



Step 2. Joins all the vertices with a line and it will form a triangle.




Question 16.

The base of an equilateral triangle with side 2a lies along the y-axis such that the mid-point of the base is at the origin. Find

the vertices of the triangle.


Answer:


Given that the base of the equilateral triangle is on the y-axis and mid-point of the base is at the origin, so its figure will be like this as shown.


So here, O(0,0) is the midpoint of the base.


An equilateral triangle has all sides equal so if O is the mid-point of the base BC, so B and C are the two vertices of the triangle. Now we have two vertices of the triangle, which is the base of equilateral triangle lying on the y-axis. Now if base in on y-axis then x-axis are as bisector of the base and so our third vertices will be on the x-axis either left or right.


So now in right ∆BOA


Pythagoras Theorem: In a right-angled triangle the square of the biggest side(hypotenuse) equals the sum of the squares of the other two sides(Perpendicular and base).


BO2 + OA2 = AB2 { By Pythagoras theorem}


a2 + OA2 = (2a) 2


OA2 = 4a2- a2


OA2 = 3a2


OA = ±a√3


So vertices of triangle are A(±a√3,0) B(0,a)and C(0,-a).



Question 17.

Let ABCD be a rectangle such that AB = 10 units and BC = 8 units. Taking AB and AD as x and y-axes respectively, find the coordinates of A, B, C and D.


Answer:

Since AB and AD both have as an endpoint, we can find the coordinate of A by finding the intersection of the two sides.


So the coordinates of A will be where the x-axis and y-axis intersect.


As we know AB lies on the x-axis so the coordinates of B can be found by using the coordinates of A and changing the x-coordinate by the measure of AB.


As we know the opposite side of a rectangle, AD and BC are congruent. Now if we have a measure of BC, we can simply find the y-coordinate of D.


Since AB and AD are the x and y-axes, A is at(0,0), B is at (10,0), C is at (10,8), and D is at (0,8).




Question 18.

ABCD is a square having a length of a side 20 units. Taking the centre of the square as the origin and x and y-axes parallel to AB and AD respectively, find the coordinates of A, B, C and D.


Answer:

Square ABCD. Center = O(0,0) Origin.


AB = BC = 20 units.


Y-coordinates of AB = = -10


Y-coordinates of AD = = 10


∴the coordinates are :-



A(-10,-10)


B(10,-10)


C(10,10)


D(-10,10)




Exercise 10.2
Question 1.

Find the distance between the following pair of points:

(0, 0), (- 5, 12)


Answer:

Given points are (0, 0) and (- 5, 12)



We need to find the distance between these two points.


We know that distance(S) between the points (x1, y1) and (x2, y2) is





⇒ S = √169


⇒ S = 13


∴ The distance between the points (0, 0) and (- 5, 12) is 13 units.



Question 2.

Find the distance between the following pair of points:

(4, 5), (- 3, 2)


Answer:

Given points are (4, 5) and (- 3, 2)



We need to find the distance between these two points.


We know that distance(S) between the points (x1, y1) and (x2, y2) is





⇒ S = √58


∴ The distance between the points (4, 5) and (- 3, 2) is √58 units.



Question 3.

Find the distance between the following pair of points:

(5, - 12), (9, - 9)


Answer:

Given points are (5, - 12) and (9, - 9)



We need to find the distance between these two points.


We know that distance(S) between the points (x1, y1) and (x2, y2) is





⇒ S = √25


⇒ S = 5


∴ The distance between the points (5, - 12) and (9, - 9) is 5 units.



Question 4.

Find the distance between the following pair of points:

(- 3, 4), (3, 0)


Answer:

Given points are (- 3, 4) and (3, 0)



We need to find the distance between these two points.


We know that distance(S) between the points (x1, y1) and (x2, y2) is





⇒ S = √52



⇒ S = 2√13


∴ The distance between the points (- 3, 4) and (3, 0) is 2√13 units.



Question 5.

Find the distance between the following pair of points:

(2, 3), (4, 1)


Answer:

Given points are (2, 3) and (4, 1)



We need to find the distance between these two points.


We know that distance(S) between the points (x1, y1) and (x2, y2) is





⇒ S = √8



⇒ S = 2√2


∴ The distance between the points (2, 3) and (4, 1) is 2√2 units.



Question 6.

Find the distance between the following pair of points:

(a, b), (- a, - b)


Answer:

Given points are (a, b) and (- a, - b)



We need to find the distance between these two points.


We know that distance(S) between the points (x1, y1) and (x2, y2) is







∴ The distance between the points (a, b) and (- a, - b) is .



Question 7.

Examine whether the points (1, - 1), (- 5, 7) and (2, 6) are equidistant from the point (- 2, 3)?


Answer:

Given that we need to show that the points (1, - 1), (- 5, 7) and (2, 6) are equidistant from the point (- 2, 3).



We know that distance between two points (x1, y1) and (x2, y2) is


Let S1 be the distance between the points (1, - 1) and (- 2, 3)






⇒ S1 = 5 ..... (1)


Let S2 be the distance between the points (- 5, 7) and (- 2, 3)






⇒ S2 = 5 ..... (2)


Let S3 be the distance between the points (2, 6) and (- 2, 3)






⇒ S3 = 5 ..... (3)


From (1), (2), and (3) we got S1 = S2 = S3 which tells us that (1, - 1), (5, 7) and (2, 5) are equidistant from (- 2, 3).



Question 8.

Find a if the distance between (a, 2) and (3, 4) is 8.


Answer:

Given that the distance between the points (a, 2) and (3, 4) is 8.



We need to find the value of a.


We know that distance(S) between the points (x1, y1) and (x2, y2) is



⇒ 82 = (a - 3)2 + (- 2)2


⇒ 64 = (a - 3)2 + 4


⇒ (a - 3)2 = 60


⇒ a – 3 = ± √60


⇒ a = 3 ± √60


∴ The values of a are 3±√60.



Question 9.

A line is of length 10 units and one of its ends is (- 2, 3). If the ordinate of the other end is 9, prove that the absicca of the other end is 6 or - 10.


Answer:

Given that the line has length of 10 units and one of its ends is (- 2, 3).



It is also given that the ordinate of the other end is 9. Let us assume the other end is (x, 9).


We know that distance(S) between the points (x1, y1) and (x2, y2) is



⇒ 10 = (x + 2)2 + (- 6)2


⇒ 100 = (x + 2)2 + 36


⇒ (x + 2)2 = 64


⇒ x + 2 = ±8


⇒ x = - 2 + 8 (or) x = - 2 - 8


⇒ x = 6 or 10


∴ Thus proved.



Question 10.

Find the value of y for which the distance between the points P(2, - 3) and Q(10, y) is 10 units.


Answer:

Given that the distance between the points P(2, - 3) and Q(10, y) is 10.



We need to find the value of y.


We know that distance(S) between the points (x1, y1) and (x2, y2) is



⇒ 102 = (- 8)2 + (3 + y)2


⇒ 100 = 64 + (3 + y)2


⇒ (3 + y)2 = 36


⇒ 3 + y = ±6


⇒ y = 3 + 6 (or) y = 3 - 6


⇒ y = 9 (or) y = - 3


∴ The values of y are 9, - 3.



Question 11.

Find the distance between the points:

(at12, 2at1) and (at22, 2at2)


Answer:

Given points are (at12, 2at1) and (at22, 2at2).



We need to find the distance between these two points.


We know that distance(S) between the points (x1, y1) and (x2, y2) is






∴ The distance between the points (at12, 2at1) and (at22, 2at2) is .



Question 12.

Find the distance between the points:

(a - b, b - a) and (a + b, a + b)


Answer:

Given points are (a - b, b - a) and (a + b, a + b).



We need to find the distance between these two points.


We know that distance(S) between the points (x1, y1) and (x2, y2) is






∴ The distance between the points (a - b, b - a) and (a + b, a + b) is .



Question 13.

Find the distance between the points:

(cosθ, sinθ) and (sinθ, cosθ)


Answer:

Given points are (cosθ, sinθ) and (sinθ, cosθ).



We need to find the distance between these two points.


We know that distance(S) between the points (x1, y1) and (x2, y2) is







∴ The distance between the points (cosθ, sinθ) and (sinθ, cosθ) is .



Question 14.

Find the point on x - axis which is equidistant from the following pair of points:

(7, 6) and (- 3, 4)


Answer:

Given points are A(7, 6) and B(- 3, 4).



We need to find a point on x - axis which is equidistant from these points.


Let us assume the point on x - axis be S(x, o).


We know that distance between the points (x1, y1) and (x2, y2) is .


From the problem,


⇒ SA = SB


⇒ SA2 = SB2


⇒ (x - 7)2 + (0 - 6)2 = (x - (- 3))2 + (0 - 4)2


⇒ (x - 7)2 + (- 6)2 = (x + 3)2 + (- 4)2


⇒ x2 - 14x + 49 + 36 = x2 + 6x + 9 + 16


⇒ 20x = 60



⇒ x = 3


∴ The point on x - axis is (3, 0).



Question 15.

Find the point on x - axis which is equidistant from the following pair of points:

(3, 2) and (- 5, - 2)


Answer:

Given points are A(3, 2) and B(- 5, - 2).



We need to find a point on x - axis which is equidistant from these points.


Let us assume the point on x - axis be S(x, o).


We know that distance between the points (x1, y1) and (x2, y2) is .


From the problem,


⇒ SA = SB


⇒ SA2 = SB2


⇒ (x - 3)2 + (0 - 2)2 = (x + 5)2 + (0 - (- 2))2


⇒ (x - 3)2 + (- 2)2 = (x + 5)2 + (2)2


⇒ x2 - 6x + 9 + 4 = x2 + 10x + 25 + 4


⇒ 16x = - 16



⇒ x = - 1


∴ The point on x - axis is (- 1, 0).



Question 16.

Find the point on x - axis which is equidistant from the following pair of points:

(2, - 5) and (- 2, 9)


Answer:

Given points are A(2, - 5) and B(- 2, 9).



We need to find a point on x - axis which is equidistant from these points.


Let us assume the point on x - axis be S(x, o).


We know that distance between the points (x1, y1) and (x2, y2) is .


From the problem,


⇒ SA = SB


⇒ SA2 = SB2


⇒ (x - 2)2 + (0 - (- 5))2 = (x - (- 2))2 + (0 - 9)2


⇒ (x - 2)2 + (5)2 = (x + 2)2 + (- 9)2


⇒ x2 - 4x + 4 + 25 = x2 + 4x + 4 + 81


⇒ 8x = - 56



⇒ x = - 7


∴ The point on x - axis is (- 7, 0).



Question 17.

Find the point on y - axis which is equidistant from point (- 5, - 2) and (3, 2).


Answer:

Given points are A(- 5, - 2) and B(3, 2).



We need to find a point on y - axis which is equidistant from these points.


Let us assume the point on y - axis be S(0, y).


We know that distance between the points (x1, y1) and (x2, y2) is .


From the problem,


⇒ SA = SB


⇒ SA2 = SB2


⇒ (0 - (- 5))2 + (y - (- 2))2 = (0 - 3)2 + (y - 2)2


⇒ (5)2 + (y + 2)2 = (- 3)2 + (y - 2)2


⇒ 25 + y2 + 4y + 4 = 9 + y2 - 4y + 4


⇒ 8y = - 16



⇒ y = - 2


∴ The point on y - axis is (0, - 2).



Question 18.

Find the point on y - axis which is equidistant from the points A(6, 5) and B(- 4, 3).


Answer:

Given points are A(6, 5) and B(- 4, 3).



We need to find a point on y - axis which is equidistant from these points.


Let us assume the point on y - axis be S(0, y).


We know that distance between the points (x1, y1) and (x2, y2) is .


From the problem,


⇒ SA = SB


⇒ SA2 = SB2


⇒ (0 - 6)2 + (y - 5)2 = (0 - (- 4))2 + (y - 3)2


⇒ (- 6)2 + (y - 5)2 = (4)2 + (y - 3)2


⇒ 36 + y2 - 10y + 25 = 16 + y2 - 6y + 9


⇒ 4y = 36



⇒ y = 9


∴ The point on y - axis is (0, 9).



Question 19.

Using distance formula, examine whether the following sets of points are collinear?

(3, 5), (1, 1), (- 2, - 5)


Answer:

Given points are A(3, 5), B(1, 1) and C(- 2, - 5).



We need to check whether these points are collinear.


We know that for three points A, B and C to be collinear, the criteria to be satisfied is AC = AB + BC.


Let us find the distances first,


We know that distance between two points (x1, y1) and (x2, y2) is







⇒ AC = 5√5 ..... (1)







⇒ AB = 2√5 ..... (2)







⇒ BC = 3√5 ..... (3)


From (1), (2), (3) we can see that AB + BC = AC.


∴ The three points are collinear.



Question 20.

Using distance formula, examine whether the following sets of points are collinear?

(5, 1), (1, - 1), (11, 4)


Answer:

Given points are A(5, 1), B(1, - 1) and C(11, 4).



We need to check whether these points are collinear.


We know that for three points A, B and C to be collinear, the criteria to be satisfied is a linear relationship between AB, BC and AC.


Let us find the distances first,


We know that distance between two points (x1, y1) and (x2, y2) is







⇒ AC = 3√5 ..... (1)







⇒ AB = 2√5 ..... (2)







⇒ BC = 5√5 ..... (3)


From (1), (2), (3) we can see that AB + AC = BC.


∴ The three points are collinear.



Question 21.

Using distance formula, examine whether the following sets of points are collinear?

(0, 0), (9, 6), (3, 2)


Answer:

Given points are A(0, 0), B(9, 6) and C(3, 2).



We need to check whether these points are collinear.


We know that for three points A, B and C to be collinear, the criteria to be satisfied is a linear relationship between AB, BC and AC.


Let us find the distances first,


We know that distance between two points (x1, y1) and (x2, y2) is





⇒ AC = √13 ..... (1)







⇒ AB = 3√13 ..... (2)







⇒ BC = 2√13 ..... (3)


From (1), (2), (3) we can see that AB = BC + AC.


∴ The three points are collinear.



Question 22.

Using distance formula, examine whether the following sets of points are collinear?

(- 1, 2), (5, 0), (2, 1)


Answer:

Given points are A(- 1, 2), B(5, 0) and C(2, 1).



We need to check whether these points are collinear.


We know that for three points A, B and C to be collinear, the criteria to be satisfied is a linear relationship between AB, BC and AC.


Let us find the distances first,


We know that distance between two points (x1, y1) and (x2, y2) is





⇒ AC = √10 ..... (1)







⇒ AB = 2√10 ..... (2)





⇒ BC = √10 ..... (3)


From (1), (2), (3) we can see that AC + BC = AB.


∴ The three points are collinear.



Question 23.

Using distance formula, examine whether the following sets of points are collinear?

(1, 5), (2, 3), (- 2, - 11)


Answer:

Given points are A(1, 5), B(2, 3) and C(- 2, - 11).



We need to check whether these points are collinear.


We know that for three points A, B and C to be collinear, the criteria to be satisfied is AC = AB + BC.


Let us find the distances first,


We know that distance between two points (x1, y1) and (x2, y2) is





⇒ AC = √265 ..... (1)





⇒ AB = √5 ..... (2)







⇒ BC = 2√53 ..... (3)


From (1), (2), (3) we can see that we cannot get any linear relationship.


∴ The three points are not collinear.



Question 24.

If A = (6, 1), B = (1, 3) and C = (x, 8), find the value of x such that AB = BC.


Answer:

Given points are A(6, 1), B(1, 3) and C(x, 8). We need to find the value of x such that AB = BC.



We know that the distance between the points (x1, y1) and (x2, y2) is


⇒ AB = BC


⇒ AB2 = BC2


⇒ (6 - 1)2 + (1 - 3)2 = (1 - x)2 + (3 - 8)2


⇒ (5)2 + (- 2)2 = (1 - x)2 + (- 5)2


⇒ 25 + 4 = 1 - 2x + x2 + 25


⇒ x2 - 2x - 3 = 0


⇒ x2 - 3x + x - 3 = 0


⇒ x(x - 3) + 1(x - 3) = 0


⇒ (x + 1)(x - 3) = 0


⇒ x + 1 = 0 (or) x - 3 = 0


⇒ x = - 1 (or) x = 3


∴ The values of x are - 1 or 3.



Question 25.

Prove that the distance between the points (a + rcosθ, b + rsinθ) and (a, b) is independent of θ.


Answer:

Given points are A(a + rcosθ, b + rsinθ) and B(a, b).



We know that the distance between the points (x1, y1) and (x2, y2) is







⇒ AB = r


We can see that AB is independent of θ.


∴ Thus proved.



Question 26.

use distance formula to show that the points (cosec2θ, 0), (0, sec2θ) and (1, 1) are collinear.


Answer:

Given points are A(cosec2θ, 0), B(0, sec2θ ) and C(1, 1).



We need to check whether these points are collinear.


We know that for three points A, B and C to be collinear, the criteria to be satisfied is AB = AC + BC.


Let us find the distances first,


We know that distance between two points (x1, y1) and (x2, y2) is







..... (1)







.... - (2)




..... (3)


Now,






⇒ AC + BC = AB


∴ The three points are collinear.



Question 27.

Using distance formula show that (3, 3) is the centre of the circle passing through the points (6, 2), (0, 4) and (4, 6). Find the radius of the circle.


Answer:

Given that circle passes through the points A(6, 2), B(0, 4), C(4, 6).



Let us assume O(x, y) be the centre of the circle.


We know that distance from the centre to any point on h circle is equal.


So, OA = OB = OC


We know that distance between two points (x1, y1) and (x2, y2) is


Now,


⇒ OA = OB


⇒ OA2 = OB2


⇒ (x - 6)2 + (y - 2)2 = (x - 0)2 + (y - 4)2


⇒ x2 - 12x + 36 + y2 - 4y + 4 = x2 + y2 - 8y + 16


⇒ 12x - 4y = 24


⇒ 3x - y = 6 ..... (1)


Now,


⇒ OB = OC


⇒ OB2 = OC2


⇒ (x - 0)2 + (y - 4)2 = (x - 4)2 + (y - 6)2


⇒ x2 + y2 - 8y + 16 = x2 - 8x + 16 + y2 - 12y + 36


⇒ 8x + 4y = 36


⇒ 2x + y = 9 .... - (2)


On solving (1) and (2), we get


⇒ x = 3 and y = 3


∴ (3, 3) is the centre of the circle.


We know radius is the distance between the centre and any point on the circle.


Let ‘r’ be the radius of the circle.





⇒ r = √10


∴ The radius of the circle is √10.



Question 28.

If the point (x, y) on the tangent is equidistant from the points (2, 3) and (6, - 1), find the relation between x and y.


Answer:

Given points are A(2, 3) and B(6, - 1). It is told that S(x, y) is equidistant from A and B.



So, we get SA = SB,


We know that distance between two points (x1, y1) and (x2, y2) is .


Now,


⇒ SA = SB


⇒ SA2 = SB2


⇒ (x - 2)2 + (y - 3)2 = (x - 6)2 + (y - (- 1))2


⇒ (x - 2)2 + (y - 3)2 = (x - 6)2 + (y + 1)2


⇒ x2 - 4x + 4 + y2 - 6y + 9 = x2 - 12x + 36 + y2 + 2y + 1


⇒ 8x - 8y = 24


⇒ x - y = 3


∴ The relation between x and y is x - y = 3.



Question 29.

Find a relation between x and y such that the point (x, y) is equidistant from points (7, 1) and (3, 5).


Answer:

Given points are A(7, 1) and B(3, 5). It is told that S(x, y) is equidistant from A and B.



So, we get SA = SB,


We know that distance between two points (x1, y1) and (x2, y2) is .


Now,


⇒ SA = SB


⇒ SA2 = SB2


⇒ (x - 7)2 + (y - 1)2 = (x - 3)2 + (y - 5)2


⇒ x2 - 14x + 49 + y2 - 2y + 1 = x2 - 6x + 9 + y2 - 10y + 25


⇒ 8x - 8y = 16


⇒ x - y = 2


∴ The relation between x and y is x - y = 2.



Question 30.

If the distances of P(x, y) from points A(3, 6) and B(- 3, 4) are equal, prove that 3x + y = 5


Answer:

Given points are A(3, 6) and B(- 3, 4). It is told that S(x, y) is equidistant from A and B.



So, we get SA = SB,


We know that distance between two points (x1, y1) and (x2, y2) is .


Now,


⇒ SA = SB


⇒ SA2 = SB2


⇒ (x - 3)2 + (y - 6)2 = (x - (- 3))2 + (y - 4)2


⇒ (x - 3)2 + (y - 6)2 = (x + 3)2 + (y - 4)2


⇒ x2 - 6x + 9 + y2 - 12y + 36 = x2 + 6x + 9 + y2 - 8y + 16


⇒ 12x + 4y = 20


⇒ 3x + y = 5


∴ Thus proved.



Question 31.

If the point (x, y) be equidistant from the points (a + b, b - a) and (a - b, a + b), prove that


Answer:

Given points are A(a + b, b - a) and B(a - b, a + b). It is told that S(x, y) is equidistant from A and B.



So, we get SA = SB,


We know that distance between two points (x1, y1) and (x2, y2) is .


Now,


⇒ SA = SB


⇒ SA2 = SB2


⇒ (x - (a + b))2 + (y - (b - a))2 = (x - (a - b))2 + (y - (a + b))2


⇒ x2 - 2(a + b)x + (a + b)2 + y2 - 2(b - a)y + (b - a)2 = x2 - 2(a - b)x + (a - b)2 + y2 - 2(a + b)y + (a + b)2


⇒ x(- 2a - 2b + 2a - 2b) = y(2b - 2a - 2a - 2b)


⇒ x(- 4b) = y(- 4a)


⇒ x(b) = y(a)



Applying componendo and dividendo,



∴ Thus proved.



Question 32.

Prove that the points (3, 4), (8, - 6) and (13, 9) are the vertices of a right angled triangle.


Answer:

Given points are A(3, 4), B(8, - 6) and C(13, 9).



Let us find the distance between sides AB, BC and CA.


We know that distance between the two points (x1, y1) and (x2, y2) is .






⇒ AB = √125





⇒ BC = √250





⇒ CA = √125


Now,



⇒ AB2 + CA2 = 125 + 125


⇒ AB2 + CA2 = 250



⇒ AB2 + CA2 = BC2


∴ The given points form a right angled isosceles triangle.



Question 33.

Determine the type (isosceles, right angled, right angled isosceles, equilateral, scalene) of the following triangles whose vertices are:

(1, 1), (- ), (- 1, - 1)


Answer:

Given points are A(1, 1), B(- √3, √3) and C(- 1, - 1).



Let us find the distance between sides AB, BC and CA.


We know that the distance between the two points (x1, y1) and (x2, y2) is














We got AB = BC = CA


∴ The given points form an equilateral triangle.



Question 34.

Determine the type (isosceles, right angled, right angled isosceles, equilateral, scalene) of the following triangles whose vertices are:

(0, 2), (7, 0), (2, 5)


Answer:

Given points are A(0, 2), B(7, 0) and C(2, 5).



Let us find the distance between sides AB, BC and CA.


We know that the distance between the points (x1, y1) and (x2, y2) is .














We got AB≠BC≠CA


∴ The given points form a scalene triangle.



Question 35.

Determine the type (isosceles, right angled, right angled isosceles, equilateral, scalene) of the following triangles whose vertices are:

(- 2, 5), (7, 10), (3, - 4)


Answer:

Given points are A(- 2, 5), B(7, 10) and C(3, - 4).



Let us find the distance between sides AB, BC and CA.


We know that the distance between the points (x1, y1) and (x2, y2) is .
















We got AB = CA


Now,



⇒ AB2 + CA2 = 106 + 106


⇒ AB2 + CA2 = 212



⇒ AB2 + CA2 = BC2


∴ The given points form a right angles isosceles triangle.



Question 36.

Determine the type (isosceles, right angled, right angled isosceles, equilateral, scalene) of the following triangles whose vertices are:

(4, 4), (3, 5), (- 1, - 1)


Answer:

Given points are A(4, 4), B(3, 5) and C(- 1, - 1).



Let us find the distance between sides AB, BC and CA.


We know that the distance between the points (x1, y1) and (x2, y2) is .















Now,



⇒ AB2 + CA2 = 2 + 50


⇒ AB2 + CA2 = 52



⇒ AB2 + CA2 = BC2


∴ The given points form a right - angled triangle.



Question 37.

Determine the type (isosceles, right angled, right angled isosceles, equilateral, scalene) of the following triangles whose vertices are:

(1, 2), (3, 0), (- 1, 0)


Answer:

Given points are A(1, 2), B(3, 0) and C(- 1, 0).



Let us find the distance between sides AB, BC and CA.


We know that the distance between the points (x1, y1) and (x2, y2) is .






⇒ AB = 4










⇒ CA = 4


We got AB = BC = CA


∴ The given points form an equilateral triangle.



Question 38.

Determine the type (isosceles, right angled, right angled isosceles, equilateral, scalene) of the following triangles whose vertices are:

(0, 6), (- 5, 3), (3, 1)


Answer:

Given points are A(0, 6), B(- 5, 3) and C(3, 1).



Let us find the distance between sides AB, BC and CA.


We know that the distance between the points (x1, y1) and (x2, y2) is .















We got AB = CA


Now,




⇒ AB2 + CA2 = 68



⇒ AB2 + CA2 = BC2


∴ The given points form a right - angled isosceles triangle.



Question 39.

Determine the type (isosceles, right angled, right angled isosceles, equilateral, scalene) of the following triangles whose vertices are:

(5, - 2), (6, 4), (7, - 2)


Answer:

Given points are A(5, - 2), B(6, 4) and C(7, - 2).



Let us find the distance between sides AB, BC and CA.


We know that the distance between the points (x1, y1) and (x2, y2) is .
















We got AB = BC≠CA


∴ The given points form an isosceles triangle.



Question 40.

If A(at2, 2at), B and C(a, 0) be any three points, show that is independent of t.


Answer:

Given points are A(at2, 2at), B and C(a, 0).


Let us find the distance between sides AB, BC and CA.


We know that distance between the two points (x1, y1) and (x2, y2) is .






⇒ AC = at2 + a








Now,






is independent of t.



Question 41.

If two vertices of an equilateral triangle be (0, 0) and (3, ), find the co - ordinates of the third vertex.


Answer:

Given that A(0, 0) and B(3, ) are two vertices of an equilateral triangle.



Let us assume C(x, y) be the third vertex of the triangle.


We have AB = BC = CA


We know that the distance between the two points (x1, y1) and (x2, y2) is .


Now,


⇒ BC = CA


⇒ BC2 = CA2


⇒ (3 - x)2 + ( - y)2 = (x - 0)2 + (y - 0)2


⇒ x2 - 6x + 9 + 3 + y2 - 2y = x2 + y2


⇒ 6x = 12 - 2y


..... - (1)


⇒ AB = BC


⇒ AB2 = BC2


⇒ (0 - 3)2 + (0 - )2 = (3 - x)2 + ( - y)2


⇒ 9 + 3 = 9 - 6x + x2 + 3 - 2y + y2


From (1)





⇒ 48y2 - 48√3y - 288 = 0


⇒ y2–√3y –6 = 0


⇒ y2 - 2√3y + √3y - 6 = 0


⇒ y(y - 2√3) + √3(y - 2√3) = 0


⇒ (y + √3)(y - 2√3) = 0


⇒ y + √3 = 0 (or) y - 2√3 = 0


⇒ y = - √3 (or) y = 2√3


From (1), for y =





⇒ x = 3


From (1), for y = 2




⇒ x = 0


∴ The third vertex of equilateral triangle is (0, 2√3) and (3, √3).



Question 42.

Find the circum - centre and circum - radius of the triangle whose vertices are (- 2, 3), (2, - 1) and (4, 0).


Answer:

Given that we need to find the circum - centre and circum - radius of the triangle whose vertices are A(- 2, 3), B(2, - 1), C(4, 0).



Let us assume O(x, y) be the Circum - centre of the circle.


We know that distance from circum - centre to any vertex is equal.


So, OA = OB = OC


We know that distance between two points (x1, y1) and (x2, y2) is


Now,


⇒ OA = OB


⇒ OA2 = OB2


⇒ (x - (- 2))2 + (y - 3)2 = (x - 2)2 + (y - (- 1))2


⇒ (x + 2)2 + (y - 3)2 = (x - 2)2 + (y + 1)2


⇒ x2 + 4x + 4 + y2 - 6y + 9 = x2 - 4x + 4 + y2 + 2y + 1


⇒ 8x - 8y = - 8


⇒ x - y = - 1 ..... (1)


Now,


⇒ OB = OC


⇒ OB2 = OC2


⇒ (x - 2)2 + (y - (- 1))2 = (x - 4)2 + (y - 0)2


⇒ (x - 2)2 + (y + 1)2 = (x - 4)2 + (y)2


⇒ x2 - 4x + 4 + y2 + 2y + 1 = x2 - 8x + 16 + y2


⇒ 4x + 2y = 11 .... - (2)


On solving (1) and (2), we get


and


is the centre of the circle.


We know radius is the distance between the centre and any point on the circle.


Let ‘r’ be the circum - radius of the circle.








∴ The radius of the circle is .



Question 43.

Find the centre of a circle passing through the points (6, - 6), (3, - 7) and (3, 3).


Answer:

Given that circle passes through the points A(6, - 6), B(3, - 7), C(3, 3).



Let us assume O(x, y) be the centre of the circle.


We know that distance from the centre to any point on h circle is equal.


So, OA = OB = OC


We know that distance between two points (x1, y1) and (x2, y2) is


Now,


⇒ OA = OB


⇒ OA2 = OB2


⇒ (x - 6)2 + (y - (- 6))2 = (x - 3)2 + (y - (- 7))2


⇒ (x - 6)2 + (y + 6)2 = (x - 3)2 + (y + 7)2


⇒ x2 - 12x + 36 + y2 + 12y + 36 = x2 - 6x + 9 + y2 + 14y + 49


⇒ 6x + 2y = 14


⇒ 3x + y = 7 ..... (1)


Now,


⇒ OB = OC


⇒ OB2 = OC2


⇒ (x - 3)2 + (y - (- 7))2 = (x - 3)2 + (y - 3)2


⇒ (x - 3)2 + (y + 7)2 = (x - 3)2 + (y - 3)2


⇒ x2 - 6x + 9 + y2 + 14y + 49 = x2 - 6x + 9 + y2 - 6y + 9


⇒ 20y = - 40


⇒ y = - 2 .... - (2)


Substituting (2) in (1), we get


⇒ x = 3


∴ (3, - 2) is the centre of the circle.


We know radius is the distance between the centre and any point on the circle.


Let ‘r’ be the radius of the circle.




⇒ r = √(9 + 16)


⇒ r = √25


⇒ r = 5


∴ The radius of the circle is 5.



Question 44.

If the line segment joining the points A(a, b) and B(c, d) subtends a right angle at the origin, show that ac + bd = 0.


Answer:

Given that the line segment joining the points A(a, b) and B(c, d) subtends a right angle at the origin O(0, 0)



So, AOB is a right angled triangle with right angle at O.


We got OA2 + OB2 = AB2 [By Pythagoras Theorem]


We know that the distance between the points (x1, y1) and (x2, y2) is .


⇒ OA2 + OB2 = AB2


⇒ (0 - a)2 + (0 - b)2 + (0 - c)2 + (0 - d)2 = (a - c)2 + (b - d)2


⇒ a2 + b2 + c2 + d2 = a2 + c2 - 2ac + b2 + d2 - 2bd


⇒ 2ac + 2bd = 0


⇒ ac + bd = 0



Question 45.

The centre of the circle is (2x - 1, 3x + 1) and radius is 10 units. Find the value of x if the circle passes through the point (- 3, - 1).


Answer:

Given that the circle has centre O(2x - 1, 3x + 1) and passes through the point A(- 3, - 1) and has a radius(r) of 10 units.



We know that the radius of the circle is the distance between the centre and any point on the circle.


So, we have r = OA


⇒ OA = 10


⇒ OA2 = 100


⇒ (2x - 1 - (- 3))2 + (3x + 1 - (- 1))2 = 100


⇒ (2x + 2)2 + (3x + 2)2 = 100


⇒ 4x2 + 8x + 4 + 9x2 + 12x + 4 = 100


⇒ 13x2 + 20x - 92 = 0


⇒ 13x2 - 26x + 46x - 92 = 0


⇒ 13x(x - 2) + 46(x - 2) = 0


⇒ (13x + 46)(x - 2) = 0


⇒ 13x + 46 = 0 (or) x - 2 = 0


⇒ 13x = - 46 (or) x = 2



∴ The values of the x are or 2.



Question 46.

Prove that the points (4, 3), (6, 4), (5, 6) and (3, 5) are the vertices of a square.


Answer:

Given points are A(4, 3), B(6, 4), C(5, 6) and D(3, 5).



We need to prove that these are the vertices of a square.


We know that in the lengths of all sides are equal and the lengths of the diagonals are equal.


Let us find the lengths of the sides.


We know that the distance between the points (x1, y1) and (x2, y2) is .


Now,





⇒ AB = √5





⇒ BC = √5





⇒ CD = √5





⇒ DA = √5


We got AB = BC = CD = DA, this may be square (or) rhombus.


Now we find the lengths of the diagonals.





⇒ AC = √10





⇒ BD = √10


We got AC = BD.


∴ The points form a square.



Question 47.

Prove that the points (4, 3), (6, 4), (5, 6) and (- 4, 4) are the vertices of a square.


Answer:

Given points are A(4, 3), B(6, 4), C(5, 6) and D(- 4, 4).



We need to prove that these are the vertices of a square.


We know that in the lengths of all sides are equal and the lengths of the diagonals are equal.


Let us find the lengths of the sides.


We know that the distance between the points (x1, y1) and (x2, y2) is .


Now,


















We got AB = BC≠CD≠DA,


∴ The points doesn’t form a square.



Question 48.

Prove that the points (3, 2), (6, 3), (7, 6), (4, 5) are the vertices of a parallelogram. Is it a rectangle?


Answer:

Given points are A(3, 2), B(6, 3), C(7, 6) and D(4, 5).



We need to prove that these are the vertices of a parallelogram.


We know that in the lengths of opposite sides are equal in a parallelogram.


Let us find the lengths of the sides.


We know that the distance between the points (x1, y1) and (x2, y2) is .


Now,





⇒ AB = √10





⇒ BC = √10





⇒ CD = √10





⇒ DA = √10


We got AB = CD and BC = DA, these are the vertices of a parallelogram.


Now we find the lengths of the diagonals.





⇒ AC = √32





⇒ BD = √8


We got AC≠BD.


∴ The points doesn’t form a rectangle.



Question 49.

Prove that the points (6, 8), (3, 7), (- 2, - 2), (1, - 1) are the vertices of a parallelogram.


Answer:

Given points are A(6, 8), B(3, 7), C(- 2, - 2) and D(1, - 1).



We need to prove that these are the vertices of a parallelogram.


We know that in the lengths of opposite sides are equal in a parallelogram and the lengths of diagonals are not equal.


Let us find the lengths of the sides.


We know that the distance between the points (x1, y1) and (x2, y2) is .


Now,


















We got AB = CD and BC = DA, these are the vertices of a parallelogram or rectangle.


Now we find the lengths of the diagonals.










We got AC≠BD.


∴ The points form a parallelogram.



Question 50.

Prove that the points (4, 8), (0, 2), (3, 0) and (7, 6) are the vertices of a rectangle.


Answer:

Given points are A(4, 8), B(0, 2), C(3, 0) and D(7, 6).



We need to prove that these are the vertices of a rectangle.


We know that in the lengths of opposite sides and lengths of diagonals are equal in a rectangle.


Let us find the lengths of the sides.


We know that the distance between the points (x1, y1) and (x2, y2) is .


Now,





⇒ AB = √52





⇒ BC = √13





⇒ CD = √52





⇒ DA = √13


We got AB = CD and BC = DA, these are the vertices of a parallelogram or a rectangle.


Now we find the lengths of the diagonals.










We got AC = BD.


∴ The points form a rectangle.



Question 51.

Show that the points A(1, 0), B(5, 3), C(2, 7) and D(- 2, 4) are the vertices of a rhombus.


Answer:

Given points are A(1, 0), B(5, 3), C(2, 7) and D(- 2, 4).



We need to prove that these are the vertices of a rhombus.


We know that in the lengths of sides are equal in a rhombus and the length of diagonals are not equal.


Let us find the lengths of the sides.


We know that the distance between the points (x1, y1) and (x2, y2) is .


Now,






⇒ AB = 5






⇒ BC = 5






⇒ CD = 5






⇒ DA = 5


We got AB = BC = CD = DA, these are the vertices of a square or a rhombus.


Now we find the lengths of the diagonals.










We got AC = BD.


∴ The points form a square not rhombus.



Question 52.

Name the type or quadrilateral formed, if any, by the following points and give reasons for your answer:

(4, 5), (7, 6), (4, 3), (1, 2)


Answer:

Given points are A(4, 5), B(7, 6), C(4, 3) and D(1, 2).



Let us find the lengths of the sides.


We know that the distance between the points (x1, y1) and (x2, y2) is .


Now,


















We got AB = CD and BC = DA, this may be a parallelogram or a rectangle.


Now we find the lengths of the diagonals.










We got AC≠BD.


∴ The points form a parallelogram.



Question 53.

Name the type or quadrilateral formed, if any, by the following points and give reasons for your answer:

(- 1, - 2), (1, 0), (- 1, 2), (- 3, 0)


Answer:

Given points are A(- 1, - 2), B(1, 0), C(- 1, 2) and D(- 3, 0).



Let us find the lengths of the sides.


We know that the distance between the points (x1, y1) and (x2, y2) is .


Now,


















We got AB = BC = CD = DA, this may be square (or) rhombus.


Now we find the lengths of the diagonals.










We got AC = BD.


∴ The points form a square.



Question 54.

Name the type or quadrilateral formed, if any, by the following points and give reasons for your answer:

(- 3, 5), (3, 1), (0, 3), (- 1, - 4)


Answer:

Given points are A(- 3, 5), B(3, 1), C(0, 3) and D(- 1, - 4).



Let us find the lengths of the sides.


We know that the distance between the points (x1, y1) and (x2, y2) is .


Now,




















We got AB≠BC≠CD≠DA, this may be a quadrilateral which is not of standard shape.


Now we find the lengths of the diagonals.












⇒ AC + BC = AB


We got points ABC are collinear.


∴ The points doesn’t form a quadrilateral.



Question 55.

Two opposite vertices of a square are (- 1, 2) and (3, 2). Find the coordinates of other two vertices.


Answer:

Given that A(- 1, 2) and C(3, 2) are the opposite vertices of a square.



Let us assume the other two vertices be B(x1, y1) and D(x2, y2) and the midpoint be M


We know that midpoint of AC = Midpoint of BD = M




⇒ M = (1, 2)



⇒ x1 + x2 = 2 ..... (1)


⇒ y1 + y2 = 4 ..... (2)


We know that lengths of the sides of the square are equal.


AB = BC = CD = DA.


We know that distance between two points (x1, y1) and (x2, y2) is


⇒ AB = BC


⇒ AB2 = BC2


⇒ (x1 - (- 1))2 + (y1 - 2)2 = (x1 - 3)2 + (y1 - 2)2


⇒ x12 + 1 + 2x1 + y12 + 4 - 4y1 = x12 - 6x1 + 9 + y12 + 4 - 4y1


⇒ 8x1 = 8



⇒ x1 = 1 ..... (3)


From (1)


⇒ x2 = 2 - 1 = 1 ..... (4)


We know that points ABC form right angled isosceles triangle.


We have AB2 + BC2 = AC2


⇒ 2AB2 = (- 1 - 3)2 + (2 - 2)2


⇒ 2((1 - (- 1))2 + (y1 - 2)2) = (- 4)2 + (0)2


⇒ 2(22 + (y1 - 2)2) = 8


⇒ 4 + (y1 - 2)2 = 8


⇒ (y1 - 2)2 = 4


⇒ y1 - 2 = ±2


⇒ y1 = 2 - 2 (or) y1 = 2 + 2


⇒ y1 = 0 (or) y1 = 4


From (2)


⇒ y2 = 4 - 0


⇒ y2 = 4


⇒ y2 = 4 - 4


⇒ y2 = 0


It is clear that the other two points are (1, 0) and (1, 4).


∴ The other two points are (1, 0) and (1, 4).



Question 56.

If ABCD be a rectangle and P be any point in a plane of the rectangle, then prove that PA2 + PC2 = PB2 + PD2.


Answer:

[Hint: Take A as the origin and AB and AD as x and y - axis respectively. Let AB = a, AD = b]


Let us assume A as the origin (0, 0) and AB and AD as x and y axis with length a and b units.



Then we get points B to be (a, 0), D to be (0, b) and C to be (a, b).


Let us assume P(x, y) be any point in a plane of the rectangle.


We need to prove PA2 + PC2 = PB2 + PD2.


We know that distance between two points (x1, y1) and (x2, y2) is .


Let us assume L.H.S,


⇒ PA2 + PC2 = ((x - 0)2 + (y - 0)2) + ((x - a)2 + (y - b)2)


⇒ PA2 + PC2 = x2 + y2 + x2 - 2ax + a2 + y2 - 2by + b2


⇒ PA2 + PC2 = (x2 - 2ax + a2 + y2) + (x2 + y2 - 2by + b2)


⇒ PA2 + PC2 = ((x - a)2 + (y - 0)2) + ((x - 0)2 + (y - b)2)


⇒ PA2 + PC2 = PB2 + PD2


⇒ L.H.S = R.H.S


∴ Thus proved.



Question 57.

Prove, using co - ordinates that diagonals of a rectangle are equal.


Answer:

Let us assume ABCD be a rectangle with A as the origin and AB and AD as x and y - axes having lengths a and b units.



We get the vertices of the rectangle as follows.


⇒ A = (0, 0)


⇒ B = (a, 0)


⇒ C = (a, b)


⇒ D = (0, b)


We need to prove the lengths of the diagonals are equal.


i.e., AC = BD


We know that distance between two points (x1, y1) and (x2, y2) is .


Let us find the individual lengths of diagonals,



..... (1)



..... (2)


From (1) and (2), we can clearly say that AC = BD.


∴ The diagonals of a rectangle are equal.



Question 58.

Prove, using coordinates that the sum of squares of the diagonals of a rectangle is equal to the sum of squares of its sides.


Answer:

Let us assume ABCD be a rectangle with A as the origin and AB and AD as x and y - axes having lengths a and b units.



We get the vertices of the rectangle as follows.


⇒ A = (0, 0)


⇒ B = (a, 0)


⇒ C = (a, b)


⇒ D = (0, b)


We need to prove that the sum of squares of the diagonals of a rectangle is equal to the sum of squares of its sides.


i.e., AC2 + BD2 = AB2 + BC2 + CD2 + DA2


We know that distance between two points (x1, y1) and (x2, y2) is .


Assume L.H.S


⇒ AC2 + BD2 = ((0 - a)2 + (0 - b)2) + ((a - 0)2 + (0 - b)2)


⇒ AC2 + BD2 = a2 + b2 + a2 + b2


⇒ AC2 + BD2 = 2(a2 + b2) ..... - (1)


Assume R.H.S


⇒ AB2 + BC2 + CD2 + DA2 = ((0 - a)2 + (0 - 0)2) + ((a - a)2 + (0 - b)2) + ((a - 0)2 + (b - b)2) + ((0 - 0)2 + (b - 0)2)


⇒ AB2 + BC2 + CD2 + DA2 = a2 + 0 + 0 + b2 + a2 + 0 + 0 + b2


⇒ AB2 + BC2 + CD2 + DA2 = 2(a2 + b2) .... (2)


From (1) and (2), we can clearly say that,


⇒ AC2 + BD2 = AB2 + BC2 + CD2 + DA2


∴ Thus proved.




Exercise 10.3
Question 1.

Find the coordinates of the point which divides the line segment joining (2,4) and (6,8) in the ratio 1:3 internally and externally.


Answer:


Let P(x,y) be the point which divides the line segment internally.


Using the section formula for the internal division, i.e.


…(i)


Here, m1 = 1, m2 = 3


(x1, y1) = (2, 4) and (x2, y2) = (6, 8)


Putting the above values in the above formula, we get





⇒ x = 3, y = 5


Hence, (3,5) is the point which divides the line segment internally.



Now, Let Q(x,y) be the point which divides the line segment externally.


Using the section formula for the external division, i.e.


…(i)


Here, m1 = 1, m2 = 3


(x1, y1) = (2, 4) and (x2, y2) = (6, 8)


Putting the above values in the above formula, we get





⇒ x = 0, y = 2


Hence, (0,2) is the point which divides the line segment externally.



Question 2.

Find the coordinates of the point which divides the join of (-1,7) and (4,-3) internally in the ratio 2:3.


Answer:


Let P(x,y) be the point which divides the line segment internally.


Using the section formula for the internal division, i.e.


…(i)


Here, m1 = 2, m2 = 3


(x1, y1) = (-1, 7) and (x2, y2) = (4, -3)


Putting the above values in the above formula, we get





⇒ x = 1, y = 3


Hence, (1,3) is the point which divides the line segment internally.



Question 3.

Find the coordinates of the point which divides the line segment joining the points (4,-3) and (8,5) in the ratio 3:1 internally.


Answer:


Let P(x,y) be the point which divides the line segment internally.


Using the section formula for the internal division, i.e.


…(i)


Here, m1 = 3, m2 = 1


(x1, y1) = (4, -3) and (x2, y2) = (8, 5)


Putting the above values in the above formula, we get





⇒ x = 7, y = 3


Hence, (7,3) is the point which divides the line segment internally.



Question 4.

Find the coordinates of the points which trisect the line segment joining the points (2,3) and (6,5).


Answer:


Let P and Q be the points of trisection of AB, i.e. AP = PQ = QB


∴ P divides AB internally in the ratio 1: 2.


∴ the coordinates of P, by applying the section formula, are


…(i)


Here, m1 = 1, m2 = 2


(x1, y1) = (2, 3) and (x2, y2) = (6, 5)


Putting the above values in the above formula, we get





Now, Q also divides AB internally in the ratio 2: 1. So, the coordinates of Q are


…(i)


Here, m1 = 2, m2 = 1


(x1, y1) = (2, 3) and (x2, y2) = (6, 5)


Putting the above values in the above formula, we get





Therefore, the coordinates of the points of trisection of the line segment joining A and B are



Question 5.

Find the coordinates of the point of trisection of the line segment joining (1,-2) and (-3,4).


Answer:


Let P and Q be the points of trisection of AB, i.e. AP = PQ = QB


∴ P divides AB internally in the ratio 1: 2.


∴ the coordinates of P, by applying the section formula, are


…(i)


Here, m1 = 1, m2 = 2


(x1, y1) = (1, -2) and (x2, y2) = (-3, 4)


Putting the above values in the above formula, we get





Now, Q also divides AB internally in the ratio 2: 1. So, the coordinates of Q are


…(i)


Here, m1 = 2, m2 = 1


(x1, y1) = (1, -2) and (x2, y2) = (-3, 4)


Putting the above values in the above formula, we get





Therefore, the coordinates of the points of trisection of the line segment joining A and B are



Question 6.

The coordinates of A and B are (1,2) and (2,3) respectively, If P lies on AB, find the coordinates of P such that


Answer:


Given:



⇒ m1 = 4 and m2 = 3


and (x1, y1) = (1, 2) ; (x2, y2) = (2, 3)


Using the section formula for the internal division, i.e.


…(i)





Hence, the coordinates of P are



Question 7.

If A (4,-8), B (3,6) and C(5,-4) are the vertices of a ΔABC, D is the mid-point of BC and P is a point on AD joined such that , find the coordinates of P.


Answer:


Given: D is the midpoint of BC. So, BD = DC


Then the coordinates of D are




⇒ x = 4 and y = 1


So, coordinates of D are (4, 1)


Now, we have to find the coordinates of P.


Given:



⇒ m1 = 2 and m2 = 1


and (x1, y1) = (4, -8) ; (x2, y2) = (4, 1)


Using the section formula for the internal division, i.e.


…(i)





⇒ x = 4, y = -2


Hence, the coordinates of P are P(x,y) = P(4, -2)



Question 8.

If p divides the join of A (-2,-2) and B (2,-4) such that , find the coordinates of P.


Answer:


Given:





⇒ 7AP = 3AP + 3PB


⇒ 7AP – 3AP = 3PB


⇒ 4AP = 3PB



Hence, the point P divides AB in the ratio of 3:4


⇒ m1 = 3 and m2 = 4


and (x1, y1) = (-2, -2) ; (x2, y2) = (2, -4)


Using the section formula for the internal division, i.e.


…(i)





Hence, the coordinates of P are



Question 9.

A (1,4) and B (4,8) are two points. P is a point on AB such that AP = AB + BP. If AP = 10 find the coordinates of P.


Answer:


Given: AP = AB + BP and AP = 10


Firstly, we find the distance between A and B


d(A,B) = √(x2 – x1)2 + (y2 – y1)2


= √(4 – 1)2 + (8 – 4)2


= √(3)2 + (4)2


= √9 + 16


= √25


= 5


So, AB = 5


It is given that AP = AB + BP


⇒ 10 = 5 + BP


⇒ 10 – 5 = BP


⇒ BP = 5


⇒ A, B and P are collinear


and since AB = BP


⇒ B is the midpoint of AP


Let the coordinates of P = (x,y)




⇒ x + 1 = 8 and y + 4 = 16


⇒ x = 7 and y = 12


Hence, the coordinates of P are (7, 12)



Question 10.

The line segment joining A (2,3) and B(-3,5) is extended through each end by a length equal to its original length. Find the coordinates of the new ends.


Answer:


Let P and Q be the required new ends


Coordinates of P


Let AP = k


∴ AB = AP = k


and PB = AP + AB = k + k = 2k



∴ P divides AB externally in the ratio 1:2


Using the section formula for the external division, i.e.


…(i)


Here, m1 = 1, m2 = 2


(x1, y1) = (2, 3) and (x2, y2) = (-3, 5)


Putting the above values in the above formula, we get





⇒ x = 7, y = 1


∴Coordinates of P are (7, 1)


Coordinates of Q.


Q divides AB externally in the ratio 2:1


Again, Using the section formula for the external division, i.e.


…(i)


Here, m1 = 2, m2 = 1


(x1, y1) = (2, 3) and (x2, y2) = (-3, 5)


Putting the above values in the above formula, we get





∴Coordinates of Q are (-8, 7)



Question 11.

The line segment joining A(6,3) to B(-1,-4) is doubled in length by having half its length added to each end. Find the coordinates of the new ends.


Answer:


Let P and Q be the required new ends


Coordinates of P


Let AP = k


∴ AB = 2AP = 2k


and PB = AP + AB = k + 2k = 3k



∴ P divides AB externally in the ratio 1:3


Using the section formula for the external division, i.e.


…(i)


Here, m1 = 1, m2 = 3


(x1, y1) = (6, 3) and (x2, y2) = (-1, -4)


Putting the above values in the above formula, we get






∴Coordinates of P are


Coordinates of Q.


Q divides AB externally in the ratio 3:1


Again, Using the section formula for the external division, i.e.


…(i)


Here, m1 = 3, m2 = 1


(x1, y1) = (6, 3) and (x2, y2) = (-1, -4)


Putting the above values in the above formula, we get





∴Coordinates of Q are



Question 12.

The coordinates of two points A and B are (-1,4) and (5,1) respectively. Find the coordinates of the point P which lies on extended line AB such that it is three times as far from B as from A.


Answer:


Now, Let P(x,y) be the point which lies on extended line AB


Using the section formula for the external division, i.e.


…(i)


Here, m1 = 1, m2 = 3


(x1, y1) = (-1, 4) and (x2, y2) = (5, 1)


Putting the above values in the above formula, we get







Question 13.

Find the distances of that point from the origin which divides the line segment joining the points (5,-4) and (3,-2) in the ration 4:3.


Answer:

Let the coordinates of the point be (x,y)


Let A = (5, -4) and B = (3, -2)


Here, the point divides the line segment in the ratio 4:3


So, m1 = 4 and m2 = 3


Using section formula,


…(i)





Hence, the coordinates of P are


Now, the distance from the origin (0,0) is







Question 14.

The coordinates of the middle points of the sides of a triangle are (1,1), (2,3) and (4,1), find the coordinates of its vertices.


Answer:


Consider a ΔABC with A(x1, y1), B(x2, y2) and C(x3, y3). If P(1, 1), Q(2, 3) and R(4, 1) are the midpoints of AB, BC, and CA. Then,


…(i)


…(ii)


…(iii)


…(iv)


…(v)


…(vi)


Adding (i), (iii) and (v), we get


x1 + x2 + x2 + x3 + x1 + x3 = 2 + 4 + 8


⇒ 2(x1 + x2 + x3) = 14


⇒ x1 + x2 + x3 = 7 …(vii)


From (i) and (vii), we get


x3 = 7 – 2 = 5


From (iii) and (vii), we get


x1 = 7 – 4 = 3


From (v) and (vii), we get


x2 = 7 – 8 = -1


Now adding (ii), (iv) and (vi), we get


y1 + y2 + y2 + y3 + y1 + y3 = 2 + 6 + 2


⇒ 2(y1 + y2 + y3) = 10


⇒ y1 + y2 + y3 = 5 …(viii)


From (ii) and (viii), we get


y3 = 5 – 2 = 3


From (iv) and (vii), we get


y1 = 5 – 6 = -1


From (vi) and (vii), we get


y2 = 5 – 2 = 3


Hence, the vertices of ΔABC are A(3, -1), B(-1, 3) and C(5, 3)



Question 15.

If the points (10,5),(8,4) and (6,6) are the mid-points of the sides of a triangle, find its vertices.


Answer:


Consider a ΔABC with A(x1, y1), B(x2, y2) and C(x3, y3). If P(10, 5), Q(8, 4) and R(6, 6) are the midpoints of AB, BC, and CA. Then,


…(i)


…(ii)


…(iii)


…(iv)


…(v)


…(vi)


Adding (i), (iii) and (v), we get


x1 + x2 + x2 + x3 + x1 + x3 = 20 + 16 + 12


⇒ 2(x1 + x2 + x3) = 48


⇒ x1 + x2 + x3 = 24 …(vii)


From (i) and (vii), we get


x3 = 24 – 20 = 4


From (iii) and (vii), we get


x1 = 24 – 16 = 8


From (v) and (vii), we get


x2 = 24 – 12 = 12


Now adding (ii), (iv) and (vi), we get


y1 + y2 + y2 + y3 + y1 + y3 = 10 + 8 + 12


⇒ 2(y1 + y2 + y3) = 30


⇒ y1 + y2 + y3 = 15 …(viii)


From (ii) and (viii), we get


y3 = 15 – 10 = 5


From (iv) and (vii), we get


y1 = 15 – 8 = 7


From (vi) and (vii), we get


y2 = 15 – 12 = 3


Hence, the vertices of ΔABC are A(8, 7), B(12, 3) and C(4, 5)



Question 16.

The mid-points of the sides of a triangle are (3,4),(4,6) and (5,7). Find the coordinates of the vertices of the triangle.


Answer:


Consider a ΔABC with A(x1, y1), B(x2, y2) and C(x3, y3). If P(3, 4), Q(4, 6) and R(5, 7) are the midpoints of AB, BC, and CA. Then,


…(i)


…(ii)


…(iii)


…(iv)


…(v)


…(vi)


Adding (i), (iii) and (v), we get


x1 + x2 + x2 + x3 + x1 + x3 = 6 + 8 + 10


⇒ 2(x1 + x2 + x3) = 24


⇒ x1 + x2 + x3 =12 …(vii)


From (i) and (vii), we get


x3 = 12 – 6 = 6


From (iii) and (vii), we get


x1 = 12 – 8 = 4


From (v) and (vii), we get


x2 = 12 – 10 = 2


Now adding (ii), (iv) and (vi), we get


y1 + y2 + y2 + y3 + y1 + y3 = 8 + 12 + 14


⇒ 2(y1 + y2 + y3) = 34


⇒ y1 + y2 + y3 = 17 …(viii)


From (ii) and (viii), we get


y3 = 17 – 8 = 9


From (iv) and (vii), we get


y1 = 17 – 12 = 5


From (vi) and (vii), we get


y2 = 17 – 14 = 3


Hence, the vertices of ΔABC are A(4, 5), B(2, 3) and C(6, 9)



Question 17.

A(1,-2) and B(2,5) are two points. The lines OA, OB are produced to C and D respectively such that OC = 2OA and OD = 2OB. Find CD.


Answer:


Given:


A(1, -2) and B(2, 5) are two points.


OC = 2OA …(i)


and OD = 2OB …(ii)


Adding (i) and (ii), we get


OC + OD = 2OA + 2OB


⇒ CD = 2[OA + OB]


⇒ CD = 2[AB] …(iii)


Now, we find the distance between A and B


d(A,B) = √(x2 – x1)2 + (y2 – y1)2


= √(2 – 1)2 + {5 – (-2)}2


= √(1)2 + (5 + 2)2


= √1 + 49


= √50


= 5√2


Putting the value in eq. (iii), we get


CD = 2 × 5√2


= 10√2



Question 18.

Find the length of the medians of the triangle whose vertices are (-1,3),(1,-1) and (5,1).


Answer:


Let the given points of a triangle be A(-1, 3), B(1, -1) and C(5,1)


Let D, E and F are the midpoints of the sides BC, CA and AB respectively.


The coordinates of D are:




D = (3, 0)


The coordinates of E are:




E = (2, 2)


The coordinates of F are:




F = (0, 1)


Now, we have to find the lengths of the medians.


d(A,D) = √(x2 – x1)2 + (y2 – y1)2


= √{3 – (-1)2} + {0 – 3}2


= √(3 + 1)2 + (-3)2


= √16 + 9


= √25


= 5 units


d(B,E) = √(x2 – x1)2 + (y2 – y1)2


= √(2 – 1)2 + {2 – (-1)}2


= √(1)2 + (2 + 1)2


= √1 + 9


= √10 units


d(C,F) = √(x2 – x1)2 + (y2 – y1)2


= √(5 – 0)2 + {1 – 1}2


= √(5)2 + (0)2


= √25


= 5 units


Hence, the length of the medians AD, BE and CF are 5, √10, 5 units respectively.



Question 19.

If A(1,5), B (-2,1) and C(4,1) be the vertices of ΔABC and the internal bisector of ∠A meets BC and D, find AD.


Answer:


Given: A(1, 5), B(-2, 1) and C(4,1) are the vertices of ΔABC


Using angle bisector theorem, which states that:


The ratio of the length of the line segment BD to the length of segment DC is equal to the ratio of the length of side AB to the length of side AC:{\displaystyle {\frac {|BD|}{|DC|}}={\frac {|AB|}{|AC|}},}






⇒ BD = DC


⇒ D is the midpoint of BC


So, the coordinates of D are:




D = (1, 1)


Now, AD = √(x2 – x1)2 + (y2 – y1)2


= √(1 – 1)2 + {5 – 1}2


= √(0)2 + (4)2


= √16


= 4 units


Hence, AD = 4 units



Question 20.

If the middle point of the line segment joining (3,4) and (k,7) is (x,y) and 2x+2y+1=0, find the value of k.


Answer:


Let P be the midpoint of the line segment joining (3, 4) and (k, 7)


So, the coordinates of P are:




Again,


2x + 2y + 1 = 0



⇒ 3 + k + 11 + 1 = 0


⇒ k + 15 = 0


⇒ k = -15



Question 21.

one end of a diameter of a circle is at (2,3) and the center is (-2,5), find the coordinates of the other end of the diameter.


Answer:


Let the coordinates of the other end be (x,y).


Since (-2, 5) is the midpoint of the line joining (2,3) and (x,y)




⇒ x + 2 = -4 and y + 3 = 10


⇒ x = -4 – 2 and y = 10 – 3


⇒ x = -6 and y = 7


Hence, the coordinates of the other end are (-6, 7)



Question 22.

Find the coordinates of a point A, where AB is the diameter of a circle whose center is (2,-3), and B is (1,4)


Answer:


Let the coordinates of the A be (x,y).


Since 2, -3) is the midpoint of the line joining (1, 4) and (x,y)




⇒ x + 1 = 4 and y + 4 = -6


⇒ x = 4 – 1 and y = -6 – 4


⇒ x = 3 and y = -10


Hence, the coordinates of A are (3, -10)



Question 23.

If the point C (-1,2) divides internally the line segment joining A (2,5) and B in the ratio 3:4. Find the coordinates of B.


Answer:


Let the coordinates of B are (x, y)


It is given that the line segment divide in the ratio 3:4


So, m1 = 3 and m2 = 4


and (x’, y’) =(-1, 2); (x1, y1) = (2,5); (x2, y2) = (x, y)


Using section formula for the internal division, we get


…(i)




⇒ 3x + 8 = -7 and 3y + 20 = 14


⇒ 3x = -7 – 8 and 3y = 14 – 20


⇒ 3x = -15 and 3y = -6


⇒ x = -5 and y = -2


Hence, the coordinates of B are (-5, -2)



Question 24.

Find the ratio in which (-8,3) divides the line segment joining the points (2,-2) and (-4,1).


Answer:

Let C(-8, 3) divides the line segment AB in the ratio m:n



Here, (x, y) = (-8, 3); (x1, y1) = (2, -2) and (x2, y2) = (-4,1)


So,


⇒ -8m -8n = -4m + 2n and 3m + 3n = m - 2n


⇒ -8m + 4m - 8n - 2n = 0 and 3m – m + 3n + 2n = 0


⇒ -4m – 10n = 0 and 2m + 5n = 0


⇒ -2m – 5n = 0 and 2m + 5n = 0


⇒ 2m + 5n = 0 and 2m + 5n = 0


⇒ 2m = -5n



Hence, the ratio is 5:2 and this negative sign shows that the division is external.



Question 25.

In what ratio does the point (-4,6) divide the line segment joining the point A(-6,10) and B (3,-8)?


Answer:

Let C(-4, 6) divides the line segment AB in the ratio m:n



Here, (x, y) = (-4, 6); (x1, y1) = (-6, 10) and (x2, y2) = (3, -8)


So,


⇒ -4m -4n = 3m - 6n and 6m + 6n = -8m + 10n


⇒ -4m – 3m – 4n + 6n = 0 and 6m + 8m + 6n – 10n = 0


⇒ -7m + 2n = 0 and 14m - 4n = 0


⇒ -7m = -2n and 14m = 4n


⇒ 7m = 2n and 7m = 2n



Hence, the ratio is 2:7 and the division is internal.



Question 26.

Find the ratio in which the line segment joining (-3,10) and (6,-8) is divided by (-1,6)


Answer:

Let C(-1, 6) divides the line segment AB in the ratio m:n



Here, (x, y) = (-1, 6); (x1, y1) = (-3, 10) and (x2, y2) = (6, -8)


So,


⇒ -m – n = 6m - 3n and 6m + 6n = -8m + 10n


⇒ -m – 6m – n + 3n = 0 and 6m + 8m + 6n – 10n = 0


⇒ -7m + 2n = 0 and 14m - 4n = 0


⇒ 2n = 7m and 4n = 14m


⇒ 7m = 2n



Hence, the ratio is 2:7 and the division is internal.



Question 27.

Find the ratio in which the line segment joining (-3,-4) and (3,5) is divided by (x,2). Also, find x.


Answer:

Let C(x, 2) divides the line segment AB in the ratio m:n



Here, (x, y) = (x, 2); (x1, y1) = (-3, -4) and (x2, y2) = (3, 5)


So,


⇒ 2m + 2n = 5m – 4n


⇒ 2m – 5m = -4n – 2n


⇒ -3m = -6n


⇒ m = 2n



Now, the ratio is 2:1


Now,



⇒ 3x = 6 – 3


⇒ 3x = 3


⇒ x = 1


Hence, the ratio is 2:1 and the division is internal and the value of x = 1



Question 28.

In what ratio does the x-axis divide the line segment joining the points (2,-3) and (5,6).


Answer:

Let the line segment A(2, -3) and B(5, 6) is divided at point P(x,0) by x-axis in ratio m:n



Here, (x, y) = (x, 0); (x1, y1) = (2, -3) and (x2, y2) = (5, 6)


So,


⇒ 0 = 6m – 3n


⇒ -6m = -3n



Hence, the ratio is 1:2 and the division is internal.



Question 29.

Find the ratio in which the line segment joining A(1,-5) and B(-4,5) is divided by the x-axis. Also, find the coordinates of the point of division.


Answer:

Let the line segment A(1, -5) and B(-4, 5) is divided at point P(x,0) by x-axis in ratio m:n



Here, (x, y) = (x, 0); (x1, y1) = (1, -5) and (x2, y2) = (-4, 5)


So,


⇒ 0 = 5m – 5n


⇒ 5m = 5n



Hence, the ratio is 1:1 and the division is internal.


Now,





Hence, the coordinates of the point of division is



Question 30.

Find the ratio in which the y-axis divides the line segment joining points (5,-6) and (-1,-4). Also, find the point of intersection.


Answer:

Let the line segment A(5, -6) and B(-1, -4) is divided at point P(0, y) by y-axis in ratio m:n



Here, (x, y) = (0, y); (x1, y1) = (5, -6) and (x2, y2) = (-1, -4)


So,


⇒ 0 = -m + 5n


⇒ m = 5n



Hence, the ratio is 5:1 and the division is internal.


Now,





Hence, the coordinates of the point of division is



Question 31.

Find the centroid of the triangle whose vertices are (2,4), (6,4), (2,0).


Answer:

Here, x1 = 2, x2 = 6, x3 = 2


and y1 = 4, y2 = 4, y3 = 0


Let the coordinates of the centroid be(x,y)


So,





Hence, the centroid of a triangle is



Question 32.

The vertices of a triangle are at (2,2), (0,6) and (8,10). Find the coordinates of the trisection point of each median which is nearer the opposite side.


Answer:


Let (2, 2), (0, 6) and (8, 10) be the vertices A, B and C of the triangle respectively. Let AD, BE, CF be the medians


The coordinates of D are:




D = (4, 8)


The coordinates of E are:




E = (5, 6)


The coordinates of F are:




F = (1, 4)


Let P be the trisection point of the median AD which is nearer to the opposite side BC


∴ P divides DA in the ratio 1:2 internally





Let Q be the trisection point of the median BE which is nearer to the opposite side CA


∴ Q divides EB in the ratio 1:2 internally





Let R be the trisection point of the median CF which is nearer to the opposite side AB


∴ R divides FC in the ratio 1:2 internally





Therefore, Coordinates of required trisection points are



Question 33.

Two vertices of a triangle are (1,4) and (5,2). If its centroid is (0,-3), find the third vertex.


Answer:

Let the third vertex of a triangle be(x,y)


Here, x1 = 1, x2 = 5, x3 = x


and y1 = 4, y2 = 2, y3 = y


and the coordinates of the centroid is (0, -3)


We know that






⇒ 6 + x = 0 and 6 + y = -9


⇒ x = -6 and y = -15


Hence, the third vertex of a triangle is (-6, -15)



Question 34.

The coordinates of the centroid of a triangle are (√3,2), and two of its vertices are (2√3,-1) and (2√3,5). Find the third vertex of the triangle.


Answer:

Let the third vertex of a triangle be (x, y)


Here, x1 = 2√3, x2 = 2√3, x3 = x


and y1 = -1, y2 = 5, y3 = y


and the coordinates of the centroid is (√3, 2)


We know that






⇒ 4√3 + x = 3√3 and 4 + y = 6


⇒ x = -√3 and y = 2


Hence, the third vertex of a triangle is (-√3, 2)



Question 35.

Find the centroid of the triangle ABC whose vertices are A (9,2), B(1,10) and C(-7,-6). Find the coordinates of the middle points of its sides and hence find the centroid of the triangle formed by joining these middle points. Do the two triangles have the same centroid?


Answer:

The vertices of a triangle are A (9,2), B(1,10) and C(-7,-6)


Here, x1 = 9, x2 = 1, x3 = -7


and y1 = 2, y2 = 10, y3 = -6


Let the coordinates of the centroid be(x,y)


So,






= (1,2)


Hence, the centroid of a triangle is (1, 2)


Now,



Let D, E and F are the midpoints of the sides BC, CA and AB respectively.


The coordinates of D are:




D = (-3, 2)


The coordinates of E are:




E = (1, -2)


The coordinates of F are:




F = (5, 6)


Now, we find the centroid of a triangle formed by joining these middle points D, E, and F as shown in figure



Let P be the trisection point of the median AD which is nearer to the opposite side BC


∴ P divides DA in the ratio 1:2 internally





= (1, 2)


Let Q be the trisection point of the median BE which is nearer to the opposite side CA


∴ Q divides EB in the ratio 1:2 internally





= (1, 2)


Let R be the trisection point of the median CF which is nearer to the opposite side AB


∴ R divides FC in the ratio 1:2 internally





= (1, 2)


Yes, the triangle has the same centroid, i.e. (1,2)



Question 36.

If (1,2), (0,-1) and (2,-1) are the middle points of the sides of the triangle, find the coordinates of its centroid.


Answer:


Consider a ΔABC with A(x1, y1), B(x2, y2) and C(x3, y3). If P(1, 2), Q(0, -1) and R(2, -1) are the midpoints of AB, BC and CA. Then,


…(i)


…(ii)


…(iii)


…(iv)


…(v)


…(vi)


Adding (i), (iii) and (v), we get


x1 + x2 + x2 + x3 + x1 + x3 = 2 + 0 + 4


⇒ 2(x1 + x2 + x3) = 6


⇒ x1 + x2 + x3 = 3 …(vii)


From (i) and (vii), we get


x3 = 3 – 2 = 1


From (iii) and (vii), we get


x1 = 3 – 0 = 3


From (v) and (vii), we get


x2 = 3 – 4 = -1


Now adding (ii), (iv) and (vi), we get


y1 + y2 + y2 + y3 + y1 + y3 = 4 + (-2) + (-2)


⇒ 2(y1 + y2 + y3) = 0


⇒ y1 + y2 + y3 = 0 …(viii)


From (ii) and (viii), we get


y3 = 0 – 4 = -4


From (iv) and (vii), we get


y1 = 0 – (-2) = 2


From (vi) and (vii), we get


y2 = 0 – (-2) = 2


Hence, the vertices of ΔABC are A(3, 2), B(-1, 2) and C(1, -4)


Now, we have to find the centroid of a triangle


The vertices of a triangle are A(3, 2), B(-1, 2) and C(1, -4)


Here, x1 = 3, x2 = -1, x3 = 1


and y1 = 2, y2 = 2, y3 = -4


Let the coordinates of the centroid be(x,y)


So,





= (1,0)


Hence, the centroid of a triangle is (1, 0)



Question 37.

Show that A(-3,2), B(-5,-5), C(2,-3) and D(4,4) are the vertices of a rhombus.


Answer:

Note that to show that a quadrilateral is a rhombus, it is sufficient to show that


(a) ABCD is a parallelogram, i.e., AC and BD have the same midpoint.


(b) a pair of adjacent edges are equal


(c) the diagonal AC and BD are not equal.



Let A(-3, 2), B(-5,-5), C(2,-3) and D(4,4) are the vertices of a rhombus.


Coordinates of the midpoint of AC are



Coordinates of the midpoint of BD are



Thus, AC and BD have the same midpoint.


Hence, ABCD is a parallelogram


Now, using Distance Formula


d(A,B)= AB = √(-5 + 3)2 + (-5 – 2)2


⇒ AB = √(-2)2 + (-7)2


⇒ AB = √4 +49


⇒ AB = √53 units


d(B,C)= BC = √(-5 – 2)2 + (-5 + 3)2


⇒ BC = √(-7)2 + (-2)2


⇒ BC = √49 +4


⇒ BC = √53 units


d(C,D) = CD = √(4 – 2)2 + (4 + 3)2


⇒ CD = √(2)2 + (7)2


⇒ CD = √4 +49


⇒ CD = √53 units


d(A,D) = AD =√(4 + 3)2 +(4 – 2)2


⇒ AD = √(7)2 + (2)2


⇒ AD = √49 +4


⇒ AD = √53 units


Therefore, AB = BC = CD = AD = √53 units


Now, check for the diagonals


AC = √(2 + 3)2 + (-3 – 2)2


= √(5)2 + (-5)2


= √25 + 25


= √50


and


BD = √(4 + 5)2 + (4 + 5)2


⇒ BD = √(9)2 + (9)2


⇒ BD = √81 + 81


⇒ BD = √162


⇒ Diagonal AC ≠ Diagonal BD


Hence, ABCD is a rhombus.



Question 38.

Show that the point (3,2),(0,5),(-3,2) and (0,-1) are the vertices of a square.


Answer:

Note that to show that a quadrilateral is a square, it is sufficient to show that


(a) ABCD is a parallelogram, i.e., AC and BD bisect each other


(b) a pair of adjacent edges are equal


(c) the diagonal AC and BD are equal.



Let the vertices of a quadrilateral are A(3, 2), B(0,5), C(-3, 2) and D(0, -1).


Coordinates of the midpoint of AC are



Coordinates of the midpoint of BD are



Thus, AC and BD have the same midpoint.


Hence, ABCD is a parallelogram


Now, Using Distance Formula, we get


AB = √(x2 – x1)2 + (y2 – y1)2


= √[(0 – 3)2 + (5 - 2)2]


= √(-3)2 + (3)2


= √(9 + 9)


= √18 units


BC = √[(-3 – 0)2 + (2 - 5)2]


= √(-3)2 + (-3)2


= √(9 + 9)


= √18 units


Therefore, AB = BC = √18 units


Now, check for the diagonals


AC = √(-3 – 3)2 + (2 – 2)2


= √(-6)2 + (0)2


= √36


= 6 units


and


BD = √(0 - 0)2 + (-1 – 5)2


⇒ BD = √(0)2 + (-6)2


⇒ BD = √36


⇒ BD = 6 units


∴ AC = BD


Hence, ABCD is a square.



Question 39.

Prove that the points (-2,-1), (1,0),(4,3) and (1,2) are the vertices of a parallelogram.


Answer:

Note that to show that a quadrilateral is a parallelogram, it is sufficient to show that the diagonals of the quadrilateral bisect each other.



Let A(-2, -1), B(1, 0), C(4, 3) and D(1, 2) are the vertices of a parallelogram.


Let M be the midpoint of AC, then the coordinates of M are given by



Let N be the midpoint of BD, then the coordinates of N are given by



Thus, AC and BD have the same midpoint.


In other words, AC and BD bisect each other.


Hence, ABCD is a parallelogram.



Question 40.

Show that the points A(1,0), B(5,3), C(2,7) and D(-2,4) are the vertices of a rhombus.


Answer:

Note that to show that a quadrilateral is a rhombus, it is sufficient to show that


(a) ABCD is a parallelogram, i.e., AC and BD have the same midpoint.


(b) a pair of adjacent edges are equal



Let A(1, 0), B(5, 3), C(2, 7) and D(-2, 4) are the vertices of a rhombus.


Coordinates of the midpoint of AC are



Coordinates of the midpoint of BD are



Thus, AC and BD have the same midpoint.


Hence, ABCD is a parallelogram


Now, using Distance Formula


d(A,B)= AB = √(5 – 1)2 + (3 – 0)2


⇒ AB = √(4)2 + (3)2


⇒ AB = √16 + 9


⇒ AB = √25 = 5 units


d(B,C)= BC = √(2 – 5)2 + (7 – 3)2


⇒ BC = √(-3)2 + (4)2


⇒ BC = √9 + 16


⇒ BC = √25 = 5 units


Therefore, adjacent sides are equal.


Hence, ABCD is a rhombus.



Question 41.

Prove that the point (4,8), (0,2), (3,0) and (7,6) are the vertices of a rectangle.


Answer:

Note that to show that a quadrilateral is a rectangle, it is sufficient to show that


(a) ABCD is a parallelogram, i.e., AC and BD bisect each other and,


(b) the diagonal AC and BD are equal



Let A(4, 8), B(0, 2), C(3, 0) and D(7, 6) are the vertices of a rectangle.


Coordinates of the midpoint of AC are



Coordinates of the midpoint of BD are



Thus, AC and BD have the same midpoint.


Hence, ABCD is a parallelogram


Now, check for the diagonals by using the distance formula


AC = √(3 – 4)2 + (0 – 8)2


= √(-1)2 + (-8)2


= √1 + 64


= √65 units


and


BD = √(7 - 0)2 + (6 – 2)2


⇒ BD = √(7)2 + (4)2


⇒ BD = √49 + 16


⇒ BD = √65 units


∴ AC = BD


Hence, ABCD is a rectangle.



Question 42.

Prove that the points (4,3), (6,4), (5,6) and (3,5) are the vertices of a square.


Answer:

Note that to show that a quadrilateral is a square, it is sufficient to show that


(a) ABCD is a parallelogram, i.e., AC and BD bisect each other


(b) a pair of adjacent edges are equal


(c) the diagonal AC and BD are equal.



Let the vertices of a quadrilateral are A(4, 3), B(6, 4), C(5, 6) and D(3, 5).


Coordinates of the midpoint of AC are



Coordinates of the midpoint of BD are



Thus, AC and BD have the same midpoint.


Hence, ABCD is a parallelogram


Now, Using Distance Formula, we get


AB = √(x2 – x1)2 + (y2 – y1)2


= √[(6 – 4)2 + (4 - 3)2]


= √(2)2 + (1)2


= √(4 + 1)


= √5 units


BC = √[(5 – 6)2 + (6 - 4)2]


= √(-1)2 + (2)2


= √(1 + 4)


= √5 units


Therefore, AB = BC = √5 units


Now, check for the diagonals


AC = √(5 – 4)2 + (6 – 3)2


= √(1)2 + (3)2


= √1 + 9


= √10 units


and


BD = √(3 - 6)2 + (5 – 4)2


⇒ BD = √(-3)2 + (1)2


⇒ BD = √9 + 1


⇒ BD = √10 units


∴ AC = BD


Hence, ABCD is a square.



Question 43.

If (6,8), (3,7) and (-2,-2) be the coordinates of the three consecutive vertices of a parallelogram, find coordinates of the fourth vertex.


Answer:


Let the coordinates of the fourth vertex D be (x, y).


We know that diagonals of a parallelogram bisect each other.


∴ Midpoint of AC = Midpoint of BD …(i)


Coordinates of the midpoint of AC are



Coordinates of the midpoint of BD are



So, according to eq. (i), we have




⇒ 3 + x = 4 and 7 + y = 6


⇒ x = 1 and y = -1


Thus, the coordinates of the vertex D are (1, -1)



Question 44.

Three consecutive vertices of a rhombus are (5,3), (2,7) and (-2,4). Find the fourth vertex.


Answer:


Let the coordinates of the fourth vertex D be (x, y).


We know that diagonals of a rhombus bisect each other.


∴ Midpoint of AC = Midpoint of BD …(i)


Coordinates of the midpoint of AC are



Coordinates of the midpoint of BD are



So, according to eq. (i), we have




⇒ 2 + x = 3 and 7 + y = 7


⇒ x = 1 and y = 0


Thus, the coordinates of the vertex D are (1, 0)



Question 45.

A quadrilateral has the vertices at the point (-4,2), (2,6), (8,5) and (9,-7). Show that the mid-point of the sides of this quadrilateral are the vertices of a parallelogram.


Answer:


Let the vertices of quadrilateral be P(-4,2), Q(2,6), R(8,5) and S(9,-7)


Let A, B, C and D are the midpoints of PQ, QR, RS and SP respectively.


Now, since A is the midpoint of P(-4, 2) and Q(2, 6)


∴ Coordinates of A are



Coordinates of B are



Coordinates of C are



and


Coordinates of D are



Now,


we find the distance between A and B










Now, since length of opposite sides of the quadrilateral formed by the midpoints of the given quadrilateral are equal .i.e.


AB = CD and AD = BC


∴ it is a parallelogram


Hence Proved



Question 46.

If the points A(6,1), B(8,2), C(9,4) and D(p,3) are the vertices of a parallelogram taken in order, find the value of p.


Answer:


Let the points be A(6,1), B(8,2), C(9,4) and D(p,3)


We know that diagonals of parallelogram bisect each other.


∴ Midpoint of AC = Midpoint of BD …(i)


Coordinates of the midpoint of AC are



Coordinates of the midpoint of BD are



So, according to eq. (i), we have




⇒ 8 + p = 15


⇒ p = 15 – 8 = 7


Hence, the value of p is 7



Question 47.

Prove that the line segment joining the middle points of two sides of a triangle is half the third side.


Answer:


We take O as the origin and OX and OY as the x and y axis respectively.


Let BC = 2a, then B = (-a, 0) and C = (a, 0)


Let A = (b, c), if E and F are the midpoints of sides AC and AB respectively.


Coordinates of midpoint of AC are



Coordinates of the midpoint of AB are



Now, distance between F and E is


d(F,E) = √(x2 – x1)2 + (y2 – y1)2





= a …(i)


and Length of BC = 2a …(ii)


From (i) and (ii), we can say that



Hence Proved



Question 48.

If P,Q,R divide the side BC,CA and AB of ΔABC in the same ratio, prove that the centroid of the triangle ABC and PQR coincide.


Answer:


Let P, Q, R be the midpoints of sides BC, CA and AB respectively


Construct a ΔPQR by joining these three midpoints of the sides.


This is called the medial triangle


Since, PQ, QR and PR are midsegments of BC, AB and AC respectively


So,



Since the corresponding sides are proportional


∴ ΔPQR ≅ ΔABC


Now, we have to prove that the centroid of the triangle ABC and PQR coincide.



For that we must show that the medians of ΔABC pass through the midpoints of three sides of the medial triangle ΔPQR.


Since PQ is a midsegment of ΔABC,


⇒ PQ || BC, so PQ || BR.


And since QR is a midsegment of AB,


⇒ AB || QR, so QR || PB.


By definition, a quadrilateral PQRB is a parallelogram.


The medians BQ and CP are in fact the diagonals of the parallelogram PQRB.


And we know that the diagonals of a parallelogram bisect each other, so PD = DR.


In other words, D is the midpoint of PR.


In the similar manner, we can show that F and E are midpoints of RQ and PQ respectively.


Hence, the centroid of the triangle ABC and PQR coincide.




Exercise 10.4
Question 1.

Find the area of the triangle whose vertices are

(3, -4), (7, 5), (-1, 10)


Answer:

Given: (3, -4), (7, 5), (-1, 10)


Let us Assume A(x1, y1) = (3, -4)


Let us Assume B(x2, y2) = (7, 5)


Let us Assume C(x3, y3) = (-1, 10)


Area of triangle


Now,


Area of given triangle




= 46 square units



Question 2.

Find the area of the triangle whose vertices are

(-1.5, 3), (6, -2), (-3, 4)


Answer:

Given (-1.5, 3), (6, -2), (-3, 4)


Let us Assume A(x1, y1) = (-1.5, 3)


Let us Assume B(x2, y2) = (6, -2)


Let us Assume C(x3, y3) = (-3, 4)


Area of triangle


Now,


Area of given triangle




= 0 square units



Question 3.

Find the area of the triangle whose vertices are

(-5, -1), (3, -5), (5, 2)


Answer:

Given (-5, -1), (3, -5), (5, 2)


Let us Assume A(x1, y1) = (-5, -1)


Let us Assume B(x2, y2) = (3, -5)


Let us Assume C(x3, y3) = (5, 2)


Area of triangle


Now,


Area of given triangle




= 32 square units



Question 4.

Find the area of the triangle whose vertices are

(5, 2), (4, 7), (7, -4)


Answer:

Given (5, 2), (4, 7), (7, -4)


Let us Assume A(x1, y1) = (5, 2)


Let us Assume B(x2, y2) = (4, 7)


Let us Assume C(x3, y3) = (7, -4)


Area of triangle


Now,


Area of given triangle




= 2 square units



Question 5.

Find the area of the triangle whose vertices are

(2, 3), (-1, 0), (2, -4)


Answer:

Given (2, 3), (-1, 0), (2, -4)


Let us Assume A(x1, y1) = (2, 3)


Let us Assume B(x2, y2) = (-1, 0)


Let us Assume C(x3, y3) = (2, -4)


Area of triangle


Now,


Area of given triangle



Square units



Question 6.

Find the area of the triangle whose vertices are

(1, -1), (-4, 6), (-3, -5)


Answer:

Given (1, -1), (-4, 6), (-3, -5)


Let us Assume A(x1, y1) = (1, -1)


Let us Assume B(x2, y2) = (-4, 6)


Let us Assume C(x3, y3) = (-3, -5)


Area of triangle


Now,


Area of given triangle



Square units



Question 7.

Find the area of the triangle whose vertices are



Answer:

Given


Let us Assume A(x1, y1) =


Let us Assume B(x2, y2) =


Let us Assume C(x3, y3) =


Area of triangle


Now,


Area of given triangle



Square units



Question 8.

Find the area of the triangle whose vertices are

(-5, 7), (-4, -5), (4, 5)


Answer:

Given (-5, 7), (-4, -5), (4, 5)


Let us Assume A(x1, y1) = (-5, 7)


Let us Assume B(x2, y2) = (-4, -5)


Let us Assume C(x3, y3) = (4, 5)


Area of triangle


Now,


Area of given triangle




= 53 square units



Question 9.

Find the area of the quadrilateral whose vertices are

(1, 1), (7, -3), (12, 2) and (7, 21)


Answer:

Given (1, 1), (7, -3), (12, 2) and (7, 21)



Let us Assume A(x1, y1) = (1, 1)


Let us Assume B(x2, y2) = (7, -3)


Let us Assume C(x3, y3) = (12, 2)


Let us Assume D(x4, y4) = (7, 21)


Let us join Ac to from two triangles ∆ABC and ∆ACD


Now


Area of triangle


Then,


Area of given triangle ABC




= 25 square units


Area of given triangle ABC




= 107 square units


Area of quadrilateral ABCD = Area of ABC + Area of ACD


= 25 + 107


= 132 sq units.



Question 10.

Find the area of the quadrilateral whose vertices are

(-4, 5), (0, 7), (5, -5), and (-4, -2)


Answer:

Given (-4, 5), (0, 7), (5, -5), and (-4, -2)



To Find: Find the area of quadrilateral.


Let us Assume A(x1, y1) = (-4, 5)


Let us Assume B(x2, y2) = (0, 7)


Let us Assume C(x3, y3) = (5, -5)


Let us Assume D(x4, y4) = (4, -2)


Let us join Ac to from two triangles ∆ABC and ∆ACD


Now


Area of triangle


Then,


Area of given triangle ABC




Area of given triangle ABC





square units


Area of quadrilateral ABCD = Area of ABC + Area of ACD



Hence, Area of Quadrilateral ABCD sq units.



Question 11.

Find the area of the quadrilateral whose vertices are

Given (-5, 7), (-4, -5), (-1, -6) and (4, 5)


Answer:


To Find: Find the area of quadrilateral.


Let us Assume A(x1, y1) = (-5, 7)


Let us Assume B(x2, y2) = (-4, -5)


Let us Assume C(x3, y3) = (-1, -6)


Let us Assume D(x4, y4) = (4, 5)


Let us join Ac to from two triangles ∆ABC and ∆ACD


Now


Area of triangle


Then,


Area of given triangle ABC





Area of given triangle ABC





= 56 square units


Area of quadrilateral ABCD = Area of ABC + Area of ACD



Hence, Area sq units.



Question 12.

Find the area of the quadrilateral whose vertices are

Given (0, 0), (6, 0), (4, 3), and (0, 3)


Answer:


To Find: Find the area of quadrilateral.


Let us Assume A(x1, y1) = (0, 0)


Let us Assume B(x2, y2) = (6, 0)


Let us Assume C(x3, y3) = (4, 3)


Let us Assume D(x4, y4) = (0, 3)


Let us join Ac to from two triangles ∆ABC and ∆ACD


Now


Area of triangle


Then,


Area of given triangle ABC




= 9 Square units


Area of given triangle ABC




= 6 square units


Area of quadrilateral ABCD = Area of ABC + Area of ACD


= 9 + 6


Hence, Area = 15 sq units.



Question 13.

Find the area of the quadrilateral whose vertices are

Given (1, 0), (5, 3), (2, 7) and (-2, 4)


Answer:


To Find: Find the area of the quadrilateral.


Let us Assume A(x1, y1) = (1, 0)


Let us Assume B(x2, y2) = (5, 3)


Let us Assume C(x3, y3) = (2, 7)


Let us Assume D(x4, y4) = (-2, 4)


Let us join Ac to from two triangles ∆ABC and ∆ACD


Now


Area of triangle


Then,


Area of given triangle ABC




Square units


Area of given triangle ABC




square units


Area of quadrilateral ABCD = Area of ABC + Area of ACD



Hence, Area = 25 sq units.



Question 14.

Find the area of the quadrilateral whose vertices taken in order are (- 4, -2), (-3, -5), (3, -2) and (2, 3).


Answer:

Given: The vertices of the quadrilateral be A(-4, -2), B(-3, -5), C(3, -2) and D(2, 3).


Let join AC to form two triangles,



Now, We know that


Area of triangle


Then,


Area of triangle ABC



Square Units


Now, Area of triangle ACD



Square Units


Area of quadrilateral ABCD = Area of triangle ABC+ Area of triangle ACD




Hence, Area of quadrilateral ABCD = 28 square Units



Question 15.

A median of a triangle divides it into two triangles of equal area. Verify this result for ABC whose vertices are A(1, 2), B(2, 5), C(3, 1).


Answer:

Given a triangle whose vertices A(1, 2), B(2, 5), C(3, 1)


Let AD is the median on side BC



D will be the mid-point of segment BC. Therefore,


Coordinate of D





Area of triangle


Then,


Area of triangle ABD




sq units


Area of triangle ACD




sq units


Hence, ∆ABD = ∆ACD



Question 16.

If A, B, C are the points (-1, 5), (3, 1), (5, 7) respectively and D, E, F are the middle points of BC, CA and AB respectively, prove that ΔABC = 4ΔDEF.


Answer:

Given: ABC is a triangle with points (-1, 5), (3, 1), (5, 7)


To Find ABC=4DEF


We know that



Area of triangle


Then,


Area of triangle ABC




= 16


Now we have to find point D, E, and F.


Hence D is the midpoint of side BC then,


Coordinates of D



= (4, 4 )


Hence E is the midpoint of side AC then,


Coordinates of E



= (2, 6)


Hence F is the midpoint of side AB then,


Coordinates of F



= (1, 3)


Area of triangle


Now Area of triangle DEF




= 4


Therefore Area of ABC= 4 Area of DEF.


Hence Proved.



Question 17.

Three vertices of a triangle are A(1, 2), B(-3, 6) and C(5, 4). If D, E, and C, respectively, show that the area of triangle ABC is four times the area of triangle DEF.


Answer:

Given: ABC is a triangle with points (1, 2), (-3, 6), (5, 4)


To prove: The area of triangle ABC is four times the area of triangle DEF


We know that



Area of triangle


Then,


Area of triangle ABC




= 12


Now we have to find point D, E, F


Hence D is the midpoint of side BC then,


Coordinates of D



= (1, 5 )


Hence E is the midpoint of side AC then,


Coordinates of E



= (3, 3)


Hence F is the midpoint of side AB then,


Coordinates of F



= (-1, 4)


Area of triangle


Now Area of triangle DEF




= 3


Therefore Area of ABC = 4 Area of DEF.


Hence, Proved.



Question 18.

Find the area of the triangle formed by joining the mid-points of the sides of the triangles whose vertices are (0, -1), (2, 1) and (0, 3). Find the ratio of this area to the area of the given triangle.


Answer:

Let ABC is a triangle with points (0, -1), (2, 1), (0, 3)


To Find: Ratio of area of triangle ABC to triangle DEF


We know that



Area of triangle


Then,


Area of triangle ABC



= 4


Now we have to find point D, E, and F.


Hence D is the midpoint of side BC then,


Coordinates of D



= (1, 2 )


Hence E is the midpoint of side AC then,


Coordinates of E



= (0, 1)


Hence F is the midpoint of side AB then,


Coordinates of F



= (1, 0)


Area of triangle


Now Area of triangle DEF




= 1


Therefore Area of ABC= 4 Area of DEF.


Then, The ratio of ∆DEF and ∆ABC = 1:4



Question 19.

Find the area of a triangle ABC if the coordinates of the middle points of the sides of the triangle are (-1, - 2), (6, 1) and (3, 5).


Answer:

Given: Coordinates of middle points are D(-1, -2), E(6, 1) and F(3, 5).


To find: Area of triangle ABC



Area of triangle


Now Area of triangle DEF




square units


Hence the area of ABC is square units



Question 20.

The vertices of ΔABC are A(3, 0), B(0, 6) and (6, 9). A straight line DE divides AB and AC in the ratio 1:2 at D and E respectively, prove that


Answer:

Given, ABC is a triangle with vertices A(3, 0), B(0, 6) and C (6, 9)


To find:


We know that


Area of triangle


Now Area of triangle DEF




square units


Now, According to the question,


DE internally divides AB in the ratio 1:2 hence


Coordinates of D




= (2, 2)


E internally divides AC in the ratio 1:2 hence


Coordinates of D




= (4, 3)


Now Area of triangle ADE




square units


Therefore, Area of ∆ABC sq. units


Hence, Area of ABC = 9. Area of ADE



Question 21.

If (t, t - 2), (t + 3, t) and (t + 2, t + 2) are the vertices of a triangle, show that its area is independent of t.


Answer:

Given a triangle with vertices (t, t-2), (t+3, t) and (t+2, t+2)



Area of triangle






= 2 sq units


Hence, t is not dependant variable in the triangle.



Question 22.

If A(x, y), B(1, 2) and C(2, 1) are the vertices of a triangle of area 6 square unit, show that x+y=15 or-9


Answer:

Given: A triangle with vertices A(x, y), B(1, 2) and C (2, 1)


To find: x + y = 15 or -9


The area is 6 square units.


Area of triangle



[x + 1 – y + 2y - 4] = 12


[x + y - 3] = 12


x + y =15


Hence, Proved



Question 23.

Prove that the points (a, b+c), (b, c+a) and (c, a+b) are collinear.


Answer:

Given: A(a, b + c), B(b, c + a) and C(c, a + b)


To prove : Given points are collinear


We know the points are collinear if area(∆ABC)=0


Area of triangle


Then,


Area




= 0


Hence, Points are collinear.



Question 24.

If the points (x1y1), (x2, y2) and (x3, y3) be collinear, show that



Answer:

Given : A(x1, y1), B(x2, y2) and C(x3, y3)


We know the points are collinear if area(∆ABC)=0


Area of triangle


Then, Area


Now, Divide by



Taking common




Hence, Proved.



Question 25.

If the points (a, b), (a1, b1) and (a-a1, b-b1) are collinear, show that


Answer:

Given (a, b), (a1, b1) and (a-a1, b-b1)


We know the points are collinear if area(∆ABC)=0


Area of triangle


Then, Area



{ab + a1 b1} = ab – ab1 – a1 b + a1 b1


- ab1 - a1b = 0


-ab1 = a1b


Therofe, We can write as



Hence, Proved.



Question 26.

Show that the point (a, 0), (0, b) and (1, 1) are collinear if


Answer:

Given: (a, 0), (0, b) and (1, 1)


We know the points are collinear if area(∆ABC)=0


Area of triangle


Then, Area



ab – a – b = 0


ab = a + b


Since,



Then, a + b = ab



Hence, Proved.



Question 27.

Find the values of x if the points (2x, 2x), (3, 2x+1) and (1, 0) are collinear.


Answer:

Given (2x, 2x), (3, 2x+1) and (1, 0)


We know the points are collinear if the area(∆ABC)=0


Area of triangle


Then, Area



4x2 - 4x – 1 = 0


4x2 - 2x - 2x – 1 = 0


2x(2x - 1) - 1(2x - 1) = 0


(2x-1)(2x-1)=0


Hence,



Question 28.

Find the value of K if the points A(2, 3), B(4, k) and C(6, -3) are collinear.


Answer:

Given A(2, 3), B(4, k) and C(6, -3) are collinear


To find: Find the value of K


So, The given points are collinear, if are (∆ABC)=0


= Area of triangle =


Then,


= 2k+6 - 24 +18 - 6k = 0


= -4k = 0


Hence, K = 0



Question 29.

Find the value of K for which the points (7, -2), (5, 1), (3, k) are collinear.


Answer:

Given A(7, -2), B(5, 1) and C(3, k) are collinear


To find: Find the value of K


So, The given points are collinear, if are (∆ABC)=0


= Area of triangle =


Then,


= 7 – 7k +5k+10-9 = 0


= -2k + 8 = 0


Hence, K = 4



Question 30.

Find the value of K for which the points (8, 1), (k, -4), (2, -5) are collinear?


Answer:

Given A(8, 1), B(k, -4) and C(2, -5) are collinear


To Find: Find the value of k


So, The given points are collinear, if are (∆ABC)=0


= Area of triangle =


Then,


= 8(1)+k(-6)+2(3) = 0


= 8 - 6k + 6 = 0


= -6k = -14


Hence, K =



Question 31.

Find the value of P are the points (2, 1), (p, -1) and (-1, 3)collinear?


Answer:

Given A(2, 1), B(p, -1) and C(-1, 3) are collinear


To find: Find the value of p


So, The given points are collinear, if are (∆ABC)=0


= Area of triangle =


Then,


= 2(-4)+p(2)-2=0


= -8 +2p -2 = 0


= 2p = 10


Hence, p =



Question 32.

Show that the straight line joining the points A(0, -1) and B(15, 2) divides the line joining the points C(-1, 2)and D(4, -5) internally in the ratio 2:3.


Answer:

Given, A(0, -1) B (15, 2) divides the line on points C(-1, 2) and D(4, -5)


To Prove. Straight line divides in the ratio 2:3 internally


The equation of line


Now, Equation of line BC




⇒ 5y + 5 = x


Therefore, x – 5y = 5 ---(1)


Now, Equation of line BC



⇒ 5(y - 2) = -7(x + 1)


⇒ 5y - 10 = -7x - 7


Therefore, 7x +5y = 3 ---(2)


On solving equation (1) and (2)


X = 1 y =


Now, Point of the intersection of AB and CD is O (1,


Let us Assume that AB divides CD at O in the ratio m:n, then


x coordinate of O =


1 =


= 4m – n = m+n


= 4m – m = n+n


= 3m = 2n


= Hence Proved



Question 33.

Find the area of the triangle whose vertices are

((a+1) (a+2), (a+2)), ((a+2) (a+3), (a+3)) and ((a, 3) (a+4), (a+4))


Answer:

Given, A triangle whose vertices are A ((a+1)(a+2), (a+2))


B ((a+2)(a+3), (a+3)) and C = ((a+b)(a+4), (a+4)).


To find: Find the area of a triangle.



Since, Area of triangle =


Then, =


=


=


Common terms will be canceled out


=


Hence, = 1 sq unit



Question 34.

The point A divides the join of P(-5, 1) and Q(3, 5) in the ratio k:1. Find the two values of k for which the area of ΔABC, where B is (1, 5) and C is (7, - 2) is equal to 2 units in magnitude.


Answer:

Given: A divides the join of P(-5, 1) and Q(3, 5) in the ratio k:1


To Find: Two values of K


A divides join of PQ in the ratio k:1 hence


Coordinates of A



Now, We have A , B (1, 5), and C(7, -2)


Now, The area of ABC is equal to the magnitude 2 (Given)


Area of ABC







14k - 66 = 4k + 4


14k - 66 = -4k - 4


10k = 70


18k = 62


Hence, k = 7 and



Question 35.

The coordinates of A, B, C, D are (6, 3), (-3, 5), (4, -2) and (x, 3x) respectively. If , find x.


Answer:

Given A, B, C, and D are (6, 3), (-3, 5), (4, -2) and (x, 3x) respectively.


and ∆DBC = 2∆ABC


To find: Find x.


Since Area of triangle


Now, The area of ∆DBC




= 11x – 7 sq units


Now, The area of ∆DBC





According to question, ∆DBC = 2∆ABC



⇒ 49 = 4(11x – 7)


⇒ 49 = 44x – 28


⇒ 44x = 77



Hence,



Question 36.

If the area of the quadrilateral whose angular points taken in order are (1, 2), (-5, 6), (7, -4) and (h, -2) be zero, show that h=3.


Answer:

Given: vertices of the quadrilateral be A(1, 2), B(-5, 6), C(7, -4) and D(h, -2).


Let join AC to form two triangles,


Now, We know that


Area of triangle =


Then,


Area of triangle ABC =


=


=


= 6 sq units


Now, Area of triangle ADC =


=


=


= 3h - 15


Area of quadrilateral ABCD = Area of triangle ABC+ Area of triangle ADC


= 3h – 15 + 6


= 3h = 9


= h = 3


Hence, h is 3



Question 37.

Find the area of the triangle whose vertices A, B, C are (3, 4) (-4, 3), (8, 6) respectively and hence find the length of the perpendicular from A to BC.


Answer:

Given: A triangle whose vertices A (3, 4) B(-4, 3), C(8, 6)


To find: Find the area of Triangle and length of AD



Area of triangle =


Then,


Area of triangle ABC =


=


=


= 4 square units


We need to find the length of AD on BC


Hence, We need to find the slope first,


The slope


Now the slope of BC = = 5


If AD perpendicular BC then the slope of AD is


Therefore, The equation of is (y - y1) = m(x - x1)


(y + 1) = (x - 5)


5y + 5 = - x + 5



Question 38.

The coordinates of the centroid of a triangle and those of two of its vertices are A(3, 1), B(1, -3) and the centroid of the triangle lies on the x-axis. Find the coordinates of the third vertex C.


Answer:

Given: A(3, 1) and B(1, -3)


To find: Find the coordinate of the third vertex C.



Let Assume C = (a, b)


Centroid on C



Therefore, G(1, 0) as it lies on x-axis


⇒ 4 + a = 3


⇒ a = -1


⇒ - 2 + b = 0


⇒ b = 2


Hence, C is (-1, 2)



Question 39.

The area of a triangle is 3 square units. Two of its vertices are A(3,1), B(1,-3) and the centroid of the triangle lies on x-axis. Find the coordinates of the third vertex C.


Answer:

Given: Coordinates of Triangle are A(3, 1) and B(1, -3)


Centroid of triangle lies on x- axis.


Let the third coordinate be C(x, y)


Centroid of the triangle with vertices (x1, y1), (x2, y2), (x3, y3) is given by,


C(X, Y)


The y coordinate of centroid will be 0, as it lies opn x – axis.


Therefore,


y1 + y2 + y3 = 0


1 – 3 + y3 = 0


y3 = 2


So, C(x, y) becomes (x, 2)


We are given that the area of triangle = 3 square units.


We know that,


Area of triangle


Putting the values we get,


3


-15 – 1 + 4x = 6


4x = 22



Hence, coordinates of the third vertex are



Question 40.

The area of a parallelogram is 12 square units. Two of its vertices are the points A(-1, 3) and B (-2, 4). Find the other two vertices of the parallelogram, if the point of intersection of diagonals lies on x-axis on its positive side.


Answer:

Given: The area of a parallelogram is 12. A(-1, 3) and B(-2, 4)


To find: Find the other two vertices of the parallelogram.


Let C is (x, y) and A(-1, 3)


Since, AC is bisected at P, y coordinate ( when p = 0)


Then,


y = -3


So, Coordinate of C is (x, -3)


Now, Area of parallelogram ABCD = area of triangle ABC + Area of triangle BAD


Since, 2(Area of triangles) = area of parallelogram


We have, A(-1, 3) B(-2, 4) and C(x, -3)


Now, Area of triangle


Then,


Area of triangle ABC


6


6


12 = 5 - x


So, x = -7


Hence, Coordinate of C is (-7, -3)


In the same we will calculate for D


Let D is (x, y) and A(-2, 4)


Since, BD is bisected at Q, y coordinate ( when Q = 0)


Then,


y = -4


So, Coordinate of C is (x, -4)


We have, A(-1, 3) B(-2, 4) and C(x, -4)


Now, Area of triangle


Then,


Area of triangle ABC


6


6


12 = 6 - x


So, x = -6


Hence, Coordinate of D is (-6, -4)


Hence, C (-7, -3) and D(-6, -4)



Question 41.

Prove that the quadrilateral whose vertices are A(-2, 5), B(4, -1), C(9, 1) and D(3, 7) is a parallelogram and find its area. If E divides AC in the ratio 2:1, prove that D, E and the middle point F of BC are collinear.


Answer:

Given: Let ABCD is a quadrilateral whose vertices A(-2, 5), B(4, -1), C(9, 1) and D(3, 7).


To prove: ABCD is a parallelogram .


We have to find |AD|, |AB|, |BC|, |DC|


The distance between two sides


|AD|


= √29


|AB| =


= √72


|DC| =


= √72


|BC| =


= √29


Therefore, AB = DC and AD = BC


Hence, ABCD is a parallelogram


Now, The Area of ABCD is = |a×b| =


=


= 0i – 0j+ 42 k


|a×b| = 42


Hence The area of parallelgram is 42



Question 42.

Prove that points (-3, -1), (2, -1), (1, 1) and (-2, 1) taken in order are the vertices of a trapezium.


Answer:

Given: Points of the quadrilatreral A(-3, -1), B(2, -1), C(1, 1), and D(-2, 1).


To Prove: ABCD is a trapezium


Proof:


For proving ABCD to be a trapezium, we need to prove that two of the sides are parallel.


Therefore, AB and CD are parallel.


For proving ABCD a trapezium,


Slope of AB = Slope of CD


Slope of AB


Slope of CD


Hence, the quadrilateral is a trapezium.